Soluciones.y.propiedades

52
SOLUCIONES Y PROPIEDADES COLIGATIVAS COMPENDIOS DE QUÍMICA GENERAL PARA ESTUDIANTES DE CIENCIAS E INGENIERÍA EXIJA SU LIBRO 3.1 DEFINICIÓN DE SOLUCIÓN uy pocos materiales que encontramos en la vida diaria son sustancias puras; la mayor parte son mezclas y muchas de ellas son homogéneas, Recordemos que las mezclas homogéneas se denominan soluciones o disoluciones, por ejemplo: el aire que respiramos es una mezcla homogénea de varias sustancias gaseosas. El latón es una solución de cinc en cobre. Los océanos son una solución de muchas sustancias que se han disuelto en agua. Una solución es una mezcla homogénea de dos o mas sustancias puras, denominadas componentes de la solución, las cuales pueden ser gaseosas, líquidas o sólidas; debido a que las soluciones líquidas son las mas comunes, en esta sección enfocaremos nuestro estudio en dicho estado. Sin embargo, el estado físico de una solución lo determina a menudo el estado de su componente principal, denominado disolvente. El otro u otros componentes se denominan soluto. En la figura 3.1 es un resumen de la relación entre los materiales (materia), recuerde que las sustancias pueden ser mezcladas por procesos físicos y se pueden usar otros procesos físicos para separar la mezcla en 1 sustancias. 1 Ebbing Darrell D. Según este autor una sustancia (o sustancia pura) es una clase de materia que no puede ser separada en otras clases de materia por ningún proceso físico. M Soluto es la sustancia que se disuelve a través del disolvente en forma de moléculas o iones para formar una sola fase. En una solución puede existir mas de un soluto. Disolvente o solvente, es la sustancia donde se disuelve el llamado soluto. Generalmente es aquella que se encuentra en mayor cantidad. En toda solución existe un solo solvente. YUPI AGUA

Transcript of Soluciones.y.propiedades

Page 1: Soluciones.y.propiedades

SOLUCIONES Y PROPIEDADES COLIGATIVAS

COMPENDIOS DE QUÍMICA GENERAL PARA ESTUDIANTES DE CIENCIAS E INGENIERÍA

EXIJA SU LIBRO

3.1 DEFINICIÓN DE SOLUCIÓN

uy pocos materiales que encontramos en la vida diaria son sustancias puras; la mayor

parte son mezclas y muchas de ellas son homogéneas, Recordemos que las mezclas

homogéneas se denominan soluciones o disoluciones, por ejemplo: el aire que respiramos es

una mezcla homogénea de varias sustancias gaseosas. El latón es una solución de cinc en

cobre. Los océanos son una solución de muchas sustancias que se han disuelto en agua.

Una solución es una mezcla homogénea de dos o mas sustancias puras,

denominadas componentes de la solución, las cuales pueden ser gaseosas, líquidas

o sólidas; debido a que las soluciones líquidas son las mas comunes, en esta sección

enfocaremos nuestro estudio en dicho estado. Sin embargo, el estado físico de una solución lo

determina a menudo el estado de su componente principal, denominado disolvente. El otro u

otros componentes se denominan soluto.

En la figura 3.1 es un resumen de la relación entre los materiales (materia), recuerde que las

sustancias pueden ser mezcladas por procesos físicos y se pueden usar otros procesos físicos

para separar la mezcla en 1sustancias.

1 Ebbing Darrell D. Según este autor una sustancia (o sustancia pura) es una clase de materia que no puede ser

separada en otras clases de materia por ningún proceso físico.

M

Soluto es la sustancia que se

disuelve a través del disolvente en

forma de moléculas o iones para

formar una sola fase. En una

solución puede existir mas de un

soluto.

Disolvente o solvente, es la sustancia

donde se disuelve el llamado soluto.

Generalmente es aquella que se

encuentra en mayor cantidad. En toda

solución existe un solo solvente.

YUPI AGUA

Page 2: Soluciones.y.propiedades

SOLUCIONES Y PROPIEDADES COLIGATIVAS

COMPENDIOS DE QUÍMICA GENERAL PARA ESTUDIANTES DE CIENCIAS E INGENIERÍA

58

3.2 PSEUDO - SOLUCIÓN, SUSPENSIÓN Y

COLOIDE

uando una sustancia se disuelve o dispersa a

través de otra, formaremos una mezcla donde

encontramos tres posibilidades diferentes de

tamaños de partículas. Cada uno de estos casos

dará lugar a mezclas denominadas: solución,

coloide y suspensión, con propiedades diferentes.

La tabla 4.1 muestra una clasificación según el

tamaño partículas.

1. Cuando la sustancia se disuelve en forma de moléculas o iones entonces se denomina

solución y los componentes son denominados soluto y disolvente.

2. Cuando la sustancia se dispersa (no es soluble) y permanece firmemente dividida se

denomina coloide y sus componentes son llamados fase dispersa y fase dispersante.

3. Cuando el tamaño de la sustancia a quién se le denomina realmente micela, es mas

grande en comparación a los dos casos anteriores se denominará suspensión.

SOLUCION

COLOIDE

SUSPENSIÓN

TAMAÑO DE

MISCELAS

1-10Å

10-10000Å

>10000Å

3.2.1 Coloides hidrofílicos e hidrofóbicos

oloide es una pseudo – solución, es una mezcla heterogénea en la cual las partículas del

soluto no precipitan, se dispersan a través de una fase dispersante, se clasifican en dos

clases principales: coloides hidrofílicos y coloides hidrofóbicos. Un coloide hidrofílico (que

aman el agua) es un coloide en el cual hay una atracción fuerte entre la fase dispersa y la fase

continua (agua). Muchos de estos coloides consisten en macromoléculas (moléculas muy

grandes) dispersas en agua. Excepto por el gran tamaño de las moléculas dispersas, estos

coloides parecen soluciones normales. Un coloide hidrofóbico (que rechazan el agua), es un

coloide en el cual hay una falta de atracción entre la fase dispersa y la fase continua (agua)

Los coloides hidrofóbicos son básicamente inestables.

La tabla 3.2 muestra una clasificación de 8 tipos de coloides que son producto de la mezcla

entre los estados sólido, líquido y gaseoso, pero respetando el orden para identificar a la fase

dispersa y fase dispersante. No se incluye la mezcla gas-gas porque estos se componen de

moléculas individuales.

C

C

TABLA 3.1

Clasificación de la mezclas según el tamaño de miscela

materia

mezclas sustancias

Mezclas

heterogéneas

Mezclas

homogéneas (soluciones)

compuestos

elementos

Figura 3.1

Relación entre los materiales

Page 3: Soluciones.y.propiedades

SOLUCIONES Y PROPIEDADES COLIGATIVAS

COMPENDIOS DE QUÍMICA GENERAL PARA ESTUDIANTES DE CIENCIAS E INGENIERÍA

59

FASE

DISPERSA

FASE

DISPERSANTE

TIPO DE

COLOIDE

EJEMPLO

Gas Gas ................. ..................

Líquido Gas Aerosol líquido Niebla

Sólido Gas Aerosol sólido Humo

Gas Líquido Espuma Espuma de cerveza

Líquido Líquido Emulsión Leche

Sólido Líquido Sol y gel Pintura, jabón en agua

Gas Sólido Espuma sólida Esponja, piedra pómez

Líquido Sólido Emulsión sólida Mantequilla

Sólido Sólido Sol sólida Porcelana

Una de las formas prácticas que permite identificar a los coloides, es el hecho de aprovecha el

efecto Tyndall, donde se refleja la luz mucho mas intensa en una suspensión que en un

coloide, en una solución no hay dicho reflejo.

3.3 CLASIFICACIÓN DE LAS

SOLUCIONES

as soluciones se pueden clasificar,

atendiendo a 5 aspectos importantes:

3.3.1 Según el número de componentes

Soluciones binarias: de dos

componentes

Soluciones ternarias: de tres

componentes

Soluciones cuaternarias: de cuatro

componentes

3.3.2 Según la naturaleza del disolvente

Soluciones acuosas: el disolvente

es el agua

Soluciones orgánicas: El disolvente puede ser: benceno, acetona, etc

3.3.3 Según la naturaleza del soluto

Soluciones ácidas: presencia de sustancias ácidas: HCl, HNO3, etc.

Soluciones básicas: presencia de sustancias básicas: NaOH, Ca(OH)2

Soluciones neutras: presencia de sales neutras: NaCl, KNO3, etc.

3.3.4 De acuerdo a la cantidad de sus componentes

Soluciones diluidas: Cuando la masa del soluto es pequeña con respecto al

volumen total de la solución.

L

TABLA 3.2

Clasificación de los coloides según el estado de agregación

Fuente luminosa

Haz de luz no

visible

A B C

Haz de luz visible Haz de luz visible

Haz de luz no visible

Figura 3.2

Dispersión de un haz de luz por partículas

coloidales conocido como efecto Tyndall. La

presencia de partículas coloidales pueden

detectarse con facilidad con ayuda de un haz de

luz.

A: Oro coloidal, una solución dorada

B: Solución de NaCl

C: Humo de tabaco, Un aerosol

Page 4: Soluciones.y.propiedades

SOLUCIONES Y PROPIEDADES COLIGATIVAS

COMPENDIOS DE QUÍMICA GENERAL PARA ESTUDIANTES DE CIENCIAS E INGENIERÍA

60

Soluciones concentradas: Es aquella donde la cantidad del soluto es grande

respecto al volumen total de la solución.

Soluciones saturadas: Es aquella solución que ha alcanzado su máxima

concentración a una temperatura determinada, es decir que no admite mas soluto

porque este comienza a precipitar.

Soluciones sobresaturadas: Es cuando se disuelve mas soluto que la solución

saturada debido a ciertos factores mecánicos, por ejemplo la agitación donde se

encuentra en equilibrio inestable.

3.3.5 Según los estados de agregación de la materia

Soluciones sólidas: Donde sus componentes se hallan en el estado sólido.

Soluciones liquidas: Donde sus componentes están estado líquido.

Soluciones gaseosas: Donde sus componentes están en estado gaseoso.

Ejemplo 3.1.- Dar un ejemplo de una solución constituida por las siguientes fases a) gas-gas,

b) líquido-gas, c) líquido-líquido, d) líquido-sólido, e) sólido-sólido.

Solución:

a) Gas- gas : Aire (O2, CO2, etc en N2)

b) Líquido – gas: Soda ( CO2 (g) en H2O ())

c) Líquido – líquido: alcohol para fricciones (H2O () en i-C3H7OH ())

d) Líquido – sólido: agua de mar (NaCl, LiCl, etc en H2O ())

e) Sólido-sólido: “oro” de 14 quilates (Cu en Au)

3.4 SOLUBILIDAD

a solubilidad se define como la

cantidad de una sustancia que se

disuelve en una cantidad dada de

disolvente (por ejemplo agua) a una

temperatura dada para dar una solución

saturada; es la concentración máxima

posible. Por ejemplo, la sal de cocina

tiene una solubilidad de 36 g por 100 g de

agua; por lo tanto, al añadir 40 gramos de

NaCl, quedaran 3 g de sal sin disolverse.

3.4.1 Factores que afectan la

solubilidad

a solubilidad de un sólido es muy

sensible a los cambios de

temperatura; pero los cambios ordinarios

de presión no producen ninguna variación

significativa. La figura 3.3 muestra cómo

la temperatura afecta la solubilidad de

algunos sólidos corrientes en agua, por ejemplo la sacarosa, el nitrato de amonio y el

hidróxido de potasio ilustran muy bien este fenómeno; por el contrario, la solubilidad del

L

L

So

lub

ilid

ad

en

g s

olu

to/1

00

g a

gu

a

Temperatura en grados Celcius

100 60 50 40 30 20 10 90 80 70

200

180

80

60

40

20

160

140

120

100

KNO3

K2SO4

NaCl

KCl

KBr

KI

Figura 3.3 Efecto de la temperatura en las solubilidades de algunas

sales.

Page 5: Soluciones.y.propiedades

SOLUCIONES Y PROPIEDADES COLIGATIVAS

COMPENDIOS DE QUÍMICA GENERAL PARA ESTUDIANTES DE CIENCIAS E INGENIERÍA

61

NaCl casi no se ve afectada por un cambio de temperatura. En algunos casos es posible lograr

concentraciones superiores a las concentraciones de saturación, y entonces se tiene una

solución sobresaturada. Una solución sobresaturada se puede producir enfriando una solución

concentrada caliente.

Los compuestos iónicos son solubles en agua, alcohol metílico y amoniaco líquido; e

insolubles en octano, benceno y tetracloruro de carbono. Las moléculas de agua, alcohol

metílico y amoniaco son polares; cada molécula

posee centros positivos y negativos de carga

eléctrica como muestra la figura 3.4. Otros

líquidos polares también actúan eficientemente

en la disolución de compuestos iónicos. Las

moléculas de octano y benceno son no polares

(ver figura 3.5). Puesto que las

electronegatividades del carbono y del hidrógeno

es muy pequeña, cualquier enlace entre estos dos

átomos es no polar. El enlace entre el cloro y el

carbono sí es polar, como se puede deducir al

estudiar las 2electronegatividades de la tabla 1.3

del capítulo 1 (página 20)

Por regla general los líquidos no polares son ineficientes en la disolución de compuestos

iónicos. También encontramos que un líquido polar disolverá otros compuestos polares. El

amoniaco y el alcohol metílico se disuelven en agua. La sacarosa, sólido polar (no iónico), es

soluble en agua y alcohol metílico, e insoluble en benceno, octano y tetracloruro de carbono.

Los disolventes no polares disuelven compuestos no polares. Las grasas y aceites,

hidrocarburos no polares, se disuelven en cualquiera de los tres solventes no polares que

hemos discutido previamente, pero son

virtualmente insolubles en los tres disolventes

polares. La regla que se sigue en la elección de

disolventes dice: “ sustancias similares

disuelven sustancias similares”. Los

disolventes polares disuelven compuestos

iónicos y polares; y los no polares disuelven

compuestos no polares.

3.4.2 Solubilidad de los gases

os gases son muy poco solubles en agua y

otros líquidos corrientes. La solubilidad de

un gas puede acrecentar aumentando la presión

sobre el gas que se halla por encima de la solución. La solubilidad del oxígeno a diferentes

presiones aparece en la tabla 4.3. Es obvio que al observar la última columna, que la

solubilidad es directamente proporcional a la presión del gas. Este comportamiento es normal

para soluciones de gases en líquidos.

2 La electronegatividad de un elemento mide la tendencia relativa del átomo a atraer los electrones hacia sí

cuando se combina químicamente con otro átomo ( véase capítulo 1 ).

L

-

O

H

H

-

+

+

C

O

H

H

H

H

Figura 3.4

Algunas moléculas polares

ALCOHOL METILICO

AGUA

+

H

H H

N

AMONIACO

Figura 3.5

Algunas moléculas no polares

TETRACLORURO

DE CARBONO BENCENO

C C

C

C C

C

C

H

H

H

H

H H Cl

Cl

Cl

Cl

Page 6: Soluciones.y.propiedades

SOLUCIONES Y PROPIEDADES COLIGATIVAS

COMPENDIOS DE QUÍMICA GENERAL PARA ESTUDIANTES DE CIENCIAS E INGENIERÍA

62

La solubilidad de los gases en los

líquidos, al contrario de la de los

sólidos o líquidos, disminuye a medida

que aumenta la temperatura. Un

químico pondrá a hervir una muestra

de agua para reducir la concentración

de gases atmosféricos disueltos. Puesto

que la solubilidad del oxígeno en agua se reduce considerablemente con el calor, al vaciar

agua caliente en los ríos y lagos puede causar serio perjuicio a la vida acuática.

La solubilidad se expresa en moles del gas disueltas por

litro de agua

La presión del gas encima de la solución es 1 atm.

El efecto de la temperatura sobre la solubilidad

de los gases se muestra en la información de la

tabla 3.4. La solubilidad del CO2 es mucho mayor que la de los otros tres gases, debido a que

el CO2 reacciona con el agua para forma ácido carbónico:

CO2 + H2O H2CO3

3.5 EL PROCESO DE DISOLUCIÓN

na sustancia puede disolverse con o sin reacción química en el disolvente. El sodio

metálico se disuelve en agua desprendiendo burbujas de hidrógeno y cantidades

considerables de calor, se produce un cambio químico en el cual el H2 y el hidróxido de sodio

iónico soluble, NaOH, son los productos, la ecuación iónica total será:

2Na(s) + 2H2O 2Na+(ac) + OH

-(ac) + H2(g) (1)

El cloruro de sodio sólido, por otra parte, se disuelve en agua sin evidencia de reacción

química: NaCl(s) Na

+(ac) + Cl

-(ac) (2)

Si la primera solución (1) se evapora a sequedad, se obtiene hidróxido de sodio sólido, NaOH,

en lugar de sodio sólido metálico. Esto junto con los productos de burbujas de hidrógeno,

indica que se efectúa una reacción con el disolvente. La evaporación de la solución de cloruro

de sodio (2) permite obtener en NaCl original. La facilidad del proceso de disolución depende

de dos factores: a) el cambio de energía (reacción exotérmica o endotérmica) y b) el cambio

de desorden 3 (entropía) que acompaña al proceso, es decir el proceso de disolución procede

hacia la disminución de la energía del sistema, lo cual corresponde a un proceso exotérmico y

hacia un incremento de desorden del sistema.

3 Whitten Kennet W. define entropía como la medición del desorden de un sistema, mientras

mayor sea el desorden de un sistema, mayor será su entropía.

P [mmHg] Solubilidad, [moles O2

por litro de H2O] constante10P

m 6

175 0.000307 1.75

300 0.000500 1.67

414 0.000688 1.66

610 0.00100 1.64

760 0.00128 1.68

Gas 0 ºC 10 ºC 50 ºC 100 ºC

H2 0.000960 0.000873 0.000717 0.000715

N2 0.00105 0.000830 0.000485 0.000423

O2 0.00212 0.00170 0.000933 0.000758

CO2 0.0765 0.0533 0.0194

U

TABLA 3.3

Solubilidad del oxígeno en agua a 25 ºC

TABLA 3.4: Efecto de la temperatura sobre la

solubilidad de los gases en agua

Page 7: Soluciones.y.propiedades

SOLUCIONES Y PROPIEDADES COLIGATIVAS

COMPENDIOS DE QUÍMICA GENERAL PARA ESTUDIANTES DE CIENCIAS E INGENIERÍA

63

El primer factor que se refiere al cambio de energía se denomina calor de solución, Hsolución.

En un líquido puro, las fuerzas intermoleculares se producen todas entre moléculas similares;

al mezclar un líquido con un sólido, cada molécula experimenta fuerza procedente de otras

moléculas o iones diferentes y también de moléculas similares. Las fuerzas relativas de estas

interacciones ayudan a determinar el grado de solubilidad de un soluto en un disolvente. Las

principales interacciones que afectan la disolución de un soluto en disolvente son:

1. Atracciones soluto-soluto

2. Atracciones disolvente-disolvente

3. Atracciones disolvente-soluto

La disolución se favorece cuando el valor de los dos primeros incisos son relativamente

pequeños y el del tercero es relativamente grande. Es preciso vencer las atracciones

intermoleculares o interiónicas entre las partículas de soluto para que se disuelva, esta parte

del proceso requiere consumo de energía. La separación de moléculas de disolvente también

consume energía. Sin embargo, cuando las partículas de soluto y las moléculas del disolvente

interaccionan en la solución se libera energía y el proceso de disolución es exotérmico.

Muchos sólidos se disuelven en líquidos por procesos 4endotérmicos. La razón de que estos

procesos se produzcan es que la endotermicidad es contrarrestada por un mayor incremento en

el desorden del soluto que acompaña al proceso de disolución. Casi todos los procesos de

disolución están acompañados de un incremento de desorden tanto en el disolvente como en

el soluto. Por tanto, este factor de desorden suele ser favorable a la solubilidad.

3.6 UNIDADES DE CONCENTRACIÓN

a concentración de un soluto es la cantidad de soluto disuelto en una cantidad dada de

disolvente o de solución. La cantidad de disolvente o de solución se puede expresar en

términos de volumen o en términos de masa o de cantidad molar. Así, hay varias formas para

expresar la concentración de una solución.

La concentración de una solución suele expresarse como masa de soluto por unidad de

volumen; pero es mas significativa expresarla en términos de número de moles por unidad de

volumen. Los términos de concentración mas importantes utilizados por los químicos son:

molaridad, molalidad, normalidad y fracción molar.

3.6.1 Molaridad

a concentración molar , o molaridad [M], se define como los moles de soluto disueltos

en un litro de solución.

(3.1)

4 Proceso endotérmico es cuando en el proceso de disolución de un soluto, en el disolvente, éste absorbe energía

del medio ambiente.

L

L

solución de litro

soluto de molesM Molaridad

Page 8: Soluciones.y.propiedades

SOLUCIONES Y PROPIEDADES COLIGATIVAS

COMPENDIOS DE QUÍMICA GENERAL PARA ESTUDIANTES DE CIENCIAS E INGENIERÍA

64

Una solución acuosa de HCl que es 0.2 M, contiene 0.2 moles de HCl por litro de solución, en

la práctica si se quiere preparar esta solución, se agrega a un matraz volumétrico aforado de

1.00 una cierta cantidad de agua por ejemplo ¼ , al cual añadimos 0.2 moles de HCl ( debe

hacerse cálculos a partir de los datos que lleva la etiqueta del frasco). En seguida se llena el

matraz con agua adicional a la marca hasta el cuello y se mezcla la solución.

Ejemplo 3.2.- 0.25 g de una muestra de sulfato de cobre pentahidratado (CuSO45H2O) se

coloca en un matraz aforado de 250 cc. El matraz se llena con agua hasta la marca en el

cuello. ¿Cuál es la molaridad de la solución resultante?

Solución.- Para determinar la molaridad, se necesitan los moles de soluto.

Por consiguiente convertiremos los gramos de CuSO4.5H2O a moles:

El volumen de la solución es 0.25 litros, en consecuencia la molaridad es:

3.6.2 Equivalente - gramo de ácidos y bases

e define un equivalente – gramo de un ácido como el peso en gramos de éste que puede

producir un mol de H+ y un equivalente – gramo de una base como el peso en gramos de

ésta que pueda producir un mol de OH-. El peso equivalente o equivalente - gramo de un

ácido se obtiene dividiendo su peso fórmula o peso molecular entre el número de hidrógenos

ácidos que aporta una molécula del ácido. El peso equivalente o equivalente - gramo de una

base se obtiene dividiendo su peso fórmula o peso molecular entre el número de iones

hidróxido. A partir de definiciones anteriores, se ve que un equivalente gramo de cualquier

ácido reacciona con un equivalente – gramo de cualquier base.

No es cierto que un mol de cualquier ácido reacciona con un mol de cualquier base en una

reacción química específica. En consecuencia, según la definición de equivalentes, 1 eq-g

ácido = 1 eq-g base. Por lo general, se puede escribir la siguiente expresión para todas las

reacciones ácido base que llegan hasta el 5punto final.

Eq de ácido = eq de base o meq de ácido = meq de base (3.2)

Donde: meq = miliequivalentes, 1 eq = 1000 meq

3.6.3 Equivalente – gramo en reacciones Redox

n reacciones de oxidación – reducción se debe recordar que un agente oxidante acepta

electrones, y un agente reductor los produce. El principio de equivalente permite hacer

cálculos estequiométricos en reacciones redox. El equivalente gramo del agente oxidante y del

agente reductor es respectivamente:

5 Punto final es el punto en el cual el indicador cambia de color y se define la titulación en una reacción de

neutralización.

S

E

O5HCuSO mol 1.0g 249.54

O5HCuSO mol 1 O5HCuSO g 0.25n 24

2424

M 0.4litros 0.25

O5HCuSO mol 0.1Molaridad 24

Page 9: Soluciones.y.propiedades

SOLUCIONES Y PROPIEDADES COLIGATIVAS

COMPENDIOS DE QUÍMICA GENERAL PARA ESTUDIANTES DE CIENCIAS E INGENIERÍA

65

3.6.4 Normalidad

a concentración normal , o normalidad [N], de una solución se define como el número

de pesos equivalentes o simplemente equivalentes (eq) de soluto por litro de solución.

(3.3)

Ejemplo 3.3 ¿Cuál es la normalidad de una solución que contiene 10 g de H2SO4 en 500 ml

de solución?

Solución.- El número de moles de H2SO4 es:

El volumen de la solución es 0.5 litros, en consecuencia la normalidad es:

3.6.5 Molalidad

a concentración molal , o molalidad [m], se define como los moles de soluto disueltos

en un kilogramo de disolvente.

(3.4)

Ejemplo 3.4.- El tolueno, C6H5CH3, es un compuesto líquido similar al benceno, C6H6. Es la

materia prima para otras sustancias, entre ellas el trinitrotolueno (TNT). Encuentre la

molalidad del tolueno en una solución que contiene 5 g de tolueno en 25 g de benceno.

Solución.- El número de moles de tolueno es:

La masa del benceno que constituye el disolvente es: 0.025 kg C6H6

L

L

ganadose No.

molecular pesooxidante) (agente gEq

perdidose No.

molecular pesoreductor) (agente gEq

solución de litro

soluto de lentemiliequiva de No.NNormalidad

o solución de litro

soluto de eequivalent peso de NúmeroN Normalidad

4242

4242 SOH mol 0.102

SOH g 98

SOH mol 1SOH g 10n

N 0.4081

2

litros 0.5

SOH moles 0.102Normalidad

42

4242

SOmolH

SOgHeq

disolvente de kg

soluto de molesm Molalidad

356356

356356 CHHC mol 0.0543

CHHC g 92

CHHC de mol 1CHHC g 5

molal 2.172solvente kg

soluto mol172.2

HC kg 0.025

CHHC mol 0.0543molalidad

66

356

Page 10: Soluciones.y.propiedades

SOLUCIONES Y PROPIEDADES COLIGATIVAS

COMPENDIOS DE QUÍMICA GENERAL PARA ESTUDIANTES DE CIENCIAS E INGENIERÍA

66

t

AA

n

n

solución la de totalesmoles

A sustancia de moles

3.6.6 Formalidad

a formalidad de una solución es el número de peso fórmula en gramos de soluto por litro

de solución; el símbolo de esta unidad es F. La formalidad es muy similar a la molaridad,

por ejemplo 1 formal que se simboliza 1F del ácido clorhídrico es igual a su peso fórmula, es

decir 36.5 g HCl, por litro de disolución.

3.6.6 Fracción molar

a fracción molar de una sustancia A (A), componente de una solución se define como

los moles de sustancia A divididos entre los moles totales de solución, esto es:

(3.5)

3.7 DILUCIÓN

n la práctica de laboratorio con frecuencia se usan soluciones concentradas de

concentración conocida para preparar a partir de ellas soluciones diluidas. Por tanto, es

importante ver cómo los cambios de volumen afectan la concentración de una solución.

Cuando la concentración se expresa en una escala volumétrica, la cantidad de soluto

contenido en un volumen determinado de la solución es igual al producto del volumen por la

concentración, es decir: Cantidad de soluto disuelto = volumen concentración

Cuando se diluye una solución, el volumen aumenta y la concentración disminuye, pero la

cantidad total de soluto permanece constante. Por esta razón, dos soluciones de

concentraciones diferentes pero que contengan las mismas cantidades de soluto, están

relacionadas por la siguiente expresión:

Volumen1 Concentración1 = Volumen2 Concentración2

V1 C1 = V2 C2 (3.6)

Para reacciones de neutralización ácido – base la ecuación correspondiente y de mucha

utilidad es:

N1V1 = N2V2 (3.7)

Donde N1 y N2 son concentraciones normales y V1 y V2 son los volúmenes requeridos para la

neutralización de una base y una ácido. No obstante se usa también para diluciones.

Ejemplo 3.4.- 0.25 de una solución contiene 75 g de NaCl por litro. a) ¿A qué volumen se

debe diluir para obtener una solución de concentración 15 g de NaCl por litro?, b) ¿qué

cantidad de agua se necesita para este propósito?

Solución: a) Considerando la expresión (3.6) y despejando Volumen se tiene:

V1 C1 = V2 C2

L

L

E

Page 11: Soluciones.y.propiedades

SOLUCIONES Y PROPIEDADES COLIGATIVAS

COMPENDIOS DE QUÍMICA GENERAL PARA ESTUDIANTES DE CIENCIAS E INGENIERÍA

67

1

1

n

i

iX

b) La cantidad de agua que se requiere para esta dilución es:

(1.25 – 0.25) = 1

Ejemplo 3.5.- Calcule el volumen aproximado del agua que se debe agregar a 250 cm3 de una

solución 1.25 N para preparar una solución de concentración 0.5 N (desprecie los cambios en

volumen)

Solución: Considerando la expresión (3.7) se puede determinar inicialmente el volumen de la

nueva concentración y luego el volumen de agua requerido en la dilución de la concentración

original. N1V1 = N2V2

La cantidad de agua es: (625 – 250) cm

3 = 375 cm

3

Ejemplo 3.6.- Si una solución se prepara a partir de 1 mol de etilenglicol y 9 mol de agua, los

moles totales de la solución son 10 moles, en consecuencia la fracción molar del etilenglicol

es:

Recordemos que la suma de las fracciones de todos los componentes de una solución es igual

a 1.

(3.8)

3.8 REACCIONES QUÍMICAS EN

SOLUCIONES

omo es de conocimiento, se discutió el método

para resolver problemas en que intervienen

reacciones químicas, ahora estamos preparados para

extender el método a reacciones en solución y poder

efectuar cálculos estequiométricos a partir de un

volumen de una concentración determinada a la

cantidad de soluto presente.

Los problemas en esta reacción se basan en dos

principios:

El número de equivalentes de todas las especies en una reacción química es el mismo.

Volumen normalidad = número de equivalentes

C

Algunas estrategias:

La molaridad y la normalidad se calculan

partiendo de la densidad y del porcentaje

de composición de una solución. En

soluciones diluidas la densidad de la

solución es muy semejante a la densidad

del disolvente puro. Para calcular la

molalidad y fracción molar solo es

necesario conocer el porcentaje de

composición de la solución.

1.25

g/ 15

g/ 75 0.25

2C

1C1V

2V

33

2

112 cm 625

N 0.5

cm 250N 1.25

N

VNV

0.1moles 10

oletilenglic mol 1

Page 12: Soluciones.y.propiedades

SOLUCIONES Y PROPIEDADES COLIGATIVAS

COMPENDIOS DE QUÍMICA GENERAL PARA ESTUDIANTES DE CIENCIAS E INGENIERÍA

68

NaOH ml 154.0

ml 203.0

N

VNV

1

221

NaOH meq 135HCl meq 135solución ml 1

HCl meq 1.5solución ml 90

NaOH g 5.4NaOH geq 1

NaOH g 40

NaOH meq 1000

NaOH geq 1NaOH meq 135

Ejemplo 3.7 ¿Cuántos mililitros de NaOH 4.0 N se necesitan para neutralizar 20 ml de HCl

3.0 N?

Solución: Se puede usar el concepto de miliequivalentes, esto es:

Si: N1 = 4.0 N

N2 = 3.0 N

V2 = 20 ml HCl

V1 = ¿?

Ejemplo 3.8.- ¿Cuántos gramos de NaOH se necesitan para neutralizar 90 ml de HCl 1.5 M?

Solución: Para resolver el problema es conveniente transformar la concentración molar en

normalidad, esto es, 1 eq-g HCl = 1 mol HCl

En consecuencia:

Por tanto, en una reacción ácido base se tiene:

No eq HCl = No eq NaOH

Por consiguiente la masa de NaOH requerida es:

Ejemplo 3.9.-Encuentre el peso equivalente de KMnO4 en la siguiente reacción:

MnSO4 + KMnO4 + H2O MnO2 + H2SO4 + K2SO4

b) ¿Cuántos gramos de MnSO4 se oxidan con 50 cm3 de una solución de KMnO4 0.1 N?

Solución: Para resolver el problema efectuaremos la igualación de la ecuación química.

Mn+2

+ SO4= + K

+ + MnO4

- + H2O

0 MnO2

0 + 2H

+ + SO4

= + 2K

+ + SO4

=

2 3 e- + 4 H

+ + MnO4

- MnO2

0 + 2 H2O

0

3 2 H2O0 + Mn

+2 MnO2

0 + 4 H

+ + 2 e

-

2 MnO4- + 2 H2O + Mn

+2 5 MnO2

0 + 4 H

+

La ecuación igualada es:

3 MnSO4 + 2 KMnO4 + 2 H2O 5 MnO2 + 2 H2SO4 + K2SO4

a) el peso equivalente del permanganato de potasio es:

33 cm

meq 3.0N 3.0y

cm

meq 4.0N 4.0

N 1.5HCl mol 1

HCl geq 1HCl moles1.5

Page 13: Soluciones.y.propiedades

SOLUCIONES Y PROPIEDADES COLIGATIVAS

COMPENDIOS DE QUÍMICA GENERAL PARA ESTUDIANTES DE CIENCIAS E INGENIERÍA

69

44

4

4

4

4

4

3

43 MnSO g 0.3775MnSO mol 1

MnSO g 151

KMnO mol 2

MnSO mol 3

KMnO eq 3

KMnO mol 1

cm 1000

KMnO eq 0.1cm 50

g 52.673

g 158KMnO geq 4

b) La masa de sulfato manganoso requerido es:

3.9 PROPIEDADES COLIGATIVAS

lgunas propiedades físicas de las soluciones difieren mucho de las del disolvente puro.

Por ejemplo, el agua pura se congela a 0 ºC, pero las soluciones acuosas se congelan a

temperaturas menores. El etilenglicol se adiciona al agua de los radiadores de los automóviles,

pues es un anticongelante ya que abate (disminuye) el punto de congelación de la solución;

también eleva el punto de ebullición de la solución sobre la del agua pura, permitiendo que el

motor funcione a una temperatura mas alta.

Una solución, que conste de dos o mas componentes, carece de las propiedades físicas

constantes de una sustancia pura; estas propiedades dependen de la concentración de las

partículas del soluto y no de su naturaleza. Tales propiedades se conocen como propiedades

coligativas y éstas son: el descenso de la presión de vapor; la depresión en el punto de

congelación; la elevación en el punto de ebullición y la presión osmótica.

Las propiedades coligativas se pueden usar en la determinación de los pesos moleculares de

las sustancias disueltas y pueden dar además información acerca de las propiedades del soluto

si se conocen las propiedades del disolvente. Todas las soluciones obedecen las leyes que se

discuten en este capítulo , cuando la concentración es suficientemente baja. Por esta razón se

les conoce como leyes de las soluciones diluidas.

3.9.1 Descenso de la presión de vapor y la ley de Rault

uchos experimentos han demostrado que las soluciones que contienen líquidos no

volátiles o sólidos como solutos, siempre tienen presiones de vapor mas bajas que los

disolventes puros.

Cuando un soluto no volátil se disuelve en un líquido; parte del volumen total de la solución

es ocupada por moléculas de soluto, y por lo tanto, hay menos moléculas de disolvente por

unidad de área en la superficie. Esto ocasiona el descenso de presión de vapor del disolvente.

A

M

FIGURA 3.6

La figura A muestra la presión de vapor del agua pura, la figura B el equilibrio de dos sistemas que contienen

agua pura, en la figua C la presión manométrica (diferencia de altura) de los dos líquidos se debe al vapor de

agua pura. Esta presión es menor sobre la solución azucarada, por que hay menos moléculas de agua por

unidad de área de superficie al evaporarse.

Fig. A

Fig. B Fig. C

Page 14: Soluciones.y.propiedades

SOLUCIONES Y PROPIEDADES COLIGATIVAS

COMPENDIOS DE QUÍMICA GENERAL PARA ESTUDIANTES DE CIENCIAS E INGENIERÍA

70

t

AA

n

n

solución la de totalesmoles

A sustancia de molesX

s

s

d

d

d

d

M

m

M

m

M

m

9815.0

sM

g 5.4

18

g 9018

g 90

0.9815

El descenso de la presión de vapor asociados con solutos no volátiles y no ionizables se

resume en la ley de Rault:

“La presión de vapor de un disolvente en una solución es igual a la presión de vapor del

disolvente puro multiplicada por la fracción molar del mismo en la disolución”.

Matemáticamente se expresa:

P = Xd.Pº (4.9)

Donde: Xd, representa la fracción molar del disolvente en la solución, Pº es la presión de

vapor del disolvente puro y P es la presión de vapor del disolvente en la solución. Esta ley

permite calcular pesos moleculares.

Ejemplo 3.10.- La presión de vapor del agua pura a 25 ºC es 23.76 mmHg y la presión de

vapor de una mezcla formada por 5.4 g de soluto no volátil en 90 g de agua es 23.32 mmHg.

Determinar el peso molecular de dicho soluto.

Solución: De acuerdo a la ley de Rault:

P = Xd.Pº

9815.076.23

32.23

P

PX

od

Puesto que la fracción molar se define como:

Se puede expresar en términos de sus pesos moleculares:

reemplazando datos:

despejando Ms: Ms= 57.3 g/mol

Ejemplo 3.11.- Determine la presión de vapor a 25 ºC de una solución acuosa que consta de

10 g de sacarosa, C12H22O11 y 75 g de H2O.

Solución: De acuerdo a la ley de Rault:

P = Xd.Pº

La fracción molar del disolvente es:

s

s

d

d

d

d

d

M

m

M

m

M

m

X

993.00292.017.4

17.4

d

Page 15: Soluciones.y.propiedades

SOLUCIONES Y PROPIEDADES COLIGATIVAS

COMPENDIOS DE QUÍMICA GENERAL PARA ESTUDIANTES DE CIENCIAS E INGENIERÍA

71

0.4592.369

1.087χ 0.541

2.369

1.282χ

2.3691.0871.282n total

087.1

mol

g29

g 100n 1.282

mol

g78

g 100n

8766 HCHC

toluenobenceno

Por consiguiente La presión de vapor es: Pv = 0.993 23.76 mmHg = 243.59 mmHg

3.9.2 La ley de Rault y las soluciones de solutos volátiles

n soluciones en las cuales tanto el disolvente como el soluto tienen una presión de vapor

apreciable, se puede aplicar la ley de Rault a ambos componentes:

Para lograr cierta comprensión de tales mezclas, considere una solución ideal que contiene

dos componentes, A y B, según la ley de Rault:

PA = XA.PAº y PB = XB.PBº

La presión total del sistema es entonces, la suma de las presiones parciales de cada

componente volátil:

Ptotal = PA + PB = XA.PAº + XB.PBº (3.10)

Ejemplo 3.12.- ¿Cuál es la presión de vapor que ejerce una mezcla formada por 100 gramos

de benceno y 100 g de tolueno a 25 ºC? A dicha temperatura las presiones del vapor de

benceno y tolueno puros son, respectivamente 95.1 y 28.4 mmHg.

Solución: De acuerdo a la expresión (3.6)

Ptotal = PC6H6 + PC7H8

P total = XC6H6* PC6H6º + XC7H8 * PC7H8º

Las fracciones molares de las dos sustancias son:

La presión de vapor será: Pt = 0.549* 95.1 mmHg + 0.459 * 28.4 mmHg

Pt = 52.21 + 13.04 = 65.25 mmHg

Las soluciones pueden representarse gráficamente. La figura 3.7 muestra la ley de Rault para

una solución ideal de un soluto en

un líquido volátil. La presión de

vapor que ejerce el líquido es

proporcional a su fracción molar

en la solución.

Este diagrama se cumple para:

P = Xd.Pº

La figura 3.8 muestra la ley de

Rault aplicado a soluciones que

tienen dos componentes volátiles.

Este diagrama se cumple para:

E

Xdisolvente

PR

ES

ION

DE

VA

PO

R

DE

L D

ISO

LV

EN

TE

A B

Figura 3.7

B Xdisolvente

PR

ES

ION

DE

VA

PO

R

DE

L D

ISO

LV

EN

TE

A

Figura 3.8

III

I

II

Page 16: Soluciones.y.propiedades

SOLUCIONES Y PROPIEDADES COLIGATIVAS

COMPENDIOS DE QUÍMICA GENERAL PARA ESTUDIANTES DE CIENCIAS E INGENIERÍA

72

Ptotal = PA + PB = XA.PAº + XB.PBº

En la figura 3.8 la línea (I) es la presión parcial de A y la línea (II) es la presión parcial de B y

la línea (III) es la presión total para diferentes concentraciones de los dos líquidos volátiles.

La figura 3.9 muestra una desviación

negativa de la ley de Rault, Las

fuerzas intermoleculares existentes en

la solución son superiores a las fuerzas

intermoleculares de los componentes

puros aisladamente.

La figura 3.10 muestra la presión de

vapor superior a la presión predicha

por la ley de Rault (desviación

positiva). Las fuerzas intermoleculares

existentes en la solución son más

débiles que las de los componentes

puros independientemente.

3.9.3 LEY DE HENRY

l efecto de la presión sobre la solubilidad de un gas en un líquido se puede predecir de

manera cuantitativa. Para soluciones diluidas de un gas no reactivo en un líquido, puede

aplicarse una expresión muy similar a la ley de Rault, conocida como la ley de Henry, cuya

expresión matemática es:

Pgas = KXgas (3.11)

Donde Pgas es la presión del gas sobre la superficie de la solución y k es una constante para

un gas y un disolvente determinados a una determinada dada. Xgas representa la fracción

molar del gas disuelto; la relación es válida a bajas concentraciones y a bajas presiones.

Ejemplo 3.13.- Si 29 mg de N2 se disuelven en un litro de agua a 0 ºC y 760 mmHg de

presión de N2, ¿qué masa de N2 se disolverá en un litro de agua a 0 ºC y 5 atm de presión?

Solución: De acuerdo con la ecuación (3.11), en principio se determinará la constante de

Henry K, para luego determinar la masa de nitrógeno disuelto en las nuevas condiciones de

presión y concentración.

La fracción molar del gas es:

E

Xdisolvente A B

Figura 3.9

Xdisolvente

PR

ES

ION

DE

VA

PO

R

DE

L D

ISO

LV

EN

TE

A B

Figura 3.10

23

2

2

2

22 N mol101.036

N g 28

N mol 1

N g 1000

N g 1N mg 29

OH mol 55.56OH g 18

OH mol 1

ml 1

g 1

OH 1

ml 1000OH 1 2

2

2

22

523

N 101.864655.561

molN101.036χ

2

Page 17: Soluciones.y.propiedades

SOLUCIONES Y PROPIEDADES COLIGATIVAS

COMPENDIOS DE QUÍMICA GENERAL PARA ESTUDIANTES DE CIENCIAS E INGENIERÍA

73

La constante de Henry será:

La masa de nitrógeno disuelta en 1 litro de agua a 5 atm será:

1 litro de H2O = 55.56 mol

Resolviendo la ecuación se tiene:

3.9.4 Aumento del punto de ebullición

ecordemos que el punto de ebullición de un líquido es la temperatura a la cual la presión

de vapor se iguala a la presión aplicada en su superficie, por ejemplo la temperatura de

ebullición normal del agua sobre el nivel del mar es 100 ºC

Se ha visto que la presión de vapor de un disolvente a una temperatura dada, desciende por la

presencia de un soluto no volátil. Las soluciones de este tipo deben calentarse a temperaturas

mas altas que el disolvente puro para que su presión de vapor iguale a la presión atmosférica.

El incremento en el punto de ebullición, Te ( en relación al punto de ebullición del

disolvente puro), es directamente proporcional al número de partículas de soluto por mol de

disolvente. Sabemos que la molalidad expresa el número fijo de moles de disolvente. Así Te

es proporcional a la molalidad, como se muestra en la siguiente expresión matemática:

Te = Kem (3.12)

Te = Tf(solución) – Tf(disolvente)

Ejemplo 3.14.- Una disolución de glicocola preparada al disolver 1.62 g de sustancia en

19.62 g de agua, hierve a 100.6 ºC. Hallar el peso molecular de la glicocola. (Ke = 0.52

ºC/molal).

Solución: Datos:

Te = 100.6 ºC

Ke = 0.52 ºC/molal

De acuerdo a la relación (2.7) Te = Kem

Se puede determinar la molalidad

R

PR

ES

ION

DE

VA

PO

R

DE

L D

ISO

LV

EN

TE

4

5105.363

101.8646

atm 1

χgas

Pgas

K

5

4N 10323.910363.5

5Pgasχ

2

atm

K

510323.956.55

2

2

22

2

2

N

N

OHN

N

Nn

n

nn

n

22

2

2

22

3 N mg 145.04N g 1

N mg 1000

N mol 1

N g 28N mol105.18

Page 18: Soluciones.y.propiedades

SOLUCIONES Y PROPIEDADES COLIGATIVAS

COMPENDIOS DE QUÍMICA GENERAL PARA ESTUDIANTES DE CIENCIAS E INGENIERÍA

74

molal 1.154molalC/ 0.52

C100)(100.6

k

ΔTem

e

A partir de este dato se puede evaluar el número de moles de soluto:

soluto de moles 0.0226agua de g 19.62 agua de g 1000

soluto de moles 1.154

En consecuencia el peso molecular de soluto será:

Para hallar el peso molecular se puede considerar un segundo método propuesto por el Prof.

José Ibarz Aznárez, el cuál expresa:

Si una disolución está constituida por a gramos de soluto y A g de disolvente, y el peso

molecular del soluto es M, la molalidad de disolución es:

MA

1000am

(3.13)

Considerando la expresión (3.8), y despejando M se tiene:

g/mol 71.551.15419.62

10001.62

mA

1000aM

3.9.5 Descenso del punto de congelación

n contraste con el punto de ebullición, el punto de congelación de una solución

generalmente es mas bajo que el punto de congelación del solvente puro, como muestra la

figura 3.6. La diferencia entre estas dos temperaturas se conoce como depresión en el punto

de solidificación y se designa por Tc, y es proporcional a la concentración molal del soluto.

Esta proporcionalidad, convertida en igualdad se puede expresar por medio de la siguiente

ecuación:

Tc = Kcm (3.14)

Tc = Tf(disolvente) – Tf(solución)

La figura 3.11 muestra como un soluto no

volátil abate la presión de vapor de un

disolvente, el punto de ebullición se eleva y

el punto de congelación desciende con

respecto a los puntos correspondientes en los

disolventes puros. La magnitud de elevación

del punto de ebullición Te es menor que la

magnitud del abatimiento del punto de

congelación Tc.

E

Pre

sió

n d

e v

apo

r m

mH

g

Figura 3.11

Diagrama de fase del H2O y de una solución acuosa

Temperatura ºC

Te Tc

P

g/mol 71.68moles 0.0226

g 1.62

n

mM

s

s

Page 19: Soluciones.y.propiedades

SOLUCIONES Y PROPIEDADES COLIGATIVAS

COMPENDIOS DE QUÍMICA GENERAL PARA ESTUDIANTES DE CIENCIAS E INGENIERÍA

75

Ejemplo 3.15.- Se funde una mezcla de 0.436 g de acetanilida con 14.06 g de alcanfor de

punto de fusión 176.2 ºC. La mezcla se deja solidificar y enfriar, se reduce a polvo, se mezcla

íntimamente y se calienta. En el momento en que se termina de fundir su temperatura es de

167.0 ºC. Hallar el peso molecular de la acetanilida. (kcalcanfor = 40.0 ºC/molal)

Solución: Los datos son los siguientes:

Tc = 176.2 ºC

kc = 40.0 ºC/molal

De acuerdo a la relación (3.7) Tc = kcm

Se puede determinar la molalidad:

molal 0.23molalC/ 40.0

C)0.671(176.2

kc

ΔTcm

A partir de este dato se puede evaluar el número de moles de soluto:

soluto de moles 0.00323n

alcanfor de g 14.06 alcanfor de g 1000

soluto de moles 0.23 n

En consecuencia el peso molecular de soluto será:

g/mol 98.341moles 0.00323

g 0.436

n

mM

s

s

Para hallar el peso molecular se puede considerar la expresión (3.8), y despejando M se tiene:

SOLVENTE

PUNTO DE

EBULLICION

[ºC]

ke

[ºC/m]

PUNTO

FUSION

[ºC]

kc

[ºC/m]

H2O 100.00 0.52 0.0 1.86

C6H6 80.10 2.53 5.50 5.12

CCl4 76.80 5.02 -22.3 29.80

C2H5OH 78.40 1.22 -114.6 1.99

ClCHO 61.20 3.63 -63.50 4.68

3.9.6 Aplicaciones del aumento y descenso del punto de ebullición y fusión

respectivamente

TABLA 3.2

Algunas propiedades de disolventes comunes

g/mol 71.551.15419.62

10001.62

mA

1000aM

Page 20: Soluciones.y.propiedades

SOLUCIONES Y PROPIEDADES COLIGATIVAS

COMPENDIOS DE QUÍMICA GENERAL PARA ESTUDIANTES DE CIENCIAS E INGENIERÍA

76

Ejemplo 3.16.- El punto de ebullición de una solución de 0.402 g de naftaleno, en 26.6 g de

cloroformo, es 0.455 ºC mas alto que el del cloroformo puro. ¿Cuál es la constante

ebulloscópica del cloroformo?

Solución: A partir de la expresión (2.6) Te = Kem, se puede despejar Ke, puesto que Te

= 0.455° y la molalidad es:

molal 0.118mHC g 128

HC mol 1

CHCl kg 1

CHCl g 1000

CHCl g 26.6

HC g 0.402m

810

810

3

3

3

810

C/molal 3.86m 0.118

C 0.455

m

ΔTeKe

Ejemplo 3.17.- La presión de vapor de una solución acuosa diluida es 23.45 torr a 25 ºC,

mientras que la presión de vapor del agua pura a la misma temperatura es 23.76 torr.

Calcúlese la concentración molal del soluto, y utilícense los valores tabulados de Ke del agua

para predecir el punto de ebullición de la solución.

Solución: Puesto que la concentración molal se define como:

disolvente de kg

soluto de molesm Molalidad

se puede considerar la masa del disolvente como 1 kg = 1000 g H2O.

De acuerdo a la ley de Rault: P = Xd.Pº

987.076.23

45.23

odP

PX

Puesto que la fracción molar se define como:

Se puede expresar en términos de sus pesos moleculares:

reemplazando datos:

El número de moles del soluto, despejando de la anterior expresión es en consecuencia:

ns = 0.727 mol

s

s

d

d

d

d

M

m

M

m

M

m

987.0

t

AA

n

n

solución la de totalesmoles

A sustancia de molesX

sn18

g 100018

g 1000

0.987

Page 21: Soluciones.y.propiedades

SOLUCIONES Y PROPIEDADES COLIGATIVAS

COMPENDIOS DE QUÍMICA GENERAL PARA ESTUDIANTES DE CIENCIAS E INGENIERÍA

77

y la molalidad será:

molal 0.727agua de kg 1

soluto de mol 0.727m

El punto de ebullición de la solución es:

De acuerdo a la relación (3.7)

Te = Kem

Te = 0.52 °C/m 0.727 m = 0.38 °C

La temperatura de ebullición de la solución es:

Te = (100 + 0.38) °C = 100.38 °C

3.10 OSMOSIS Y PRESIÓN OSMÓTICA

a osmosis es el proceso espontáneo por el cual las moléculas del disolvente atraviesan una

membrana semipermeable de una solución de menor concentración de soluto hacia una

solución con mayor concentración de mayor soluto

Para definir la presión osmótica es conveniente considerar el sistema de la figura 2.12. El cual

muestra un experimento en una cámara de presión osmótica.

a) La figura 3.12 muestra el inicio de la determinación de la presión osmótica, los niveles de

solución del lado izquierdo y del disolvente del lado derecho son iguales.

b) Después del inicio del experimento, las moléculas del disolvente tienden a fluir hacia la

solución, entonces observamos rebalse de la solución como era de esperarse, como muestra la

figura 3.13.

c) Para evitar el rebalse instalemos un tubo en la cámara de la disolución; durante cierto

tiempo de iniciado el experimento, el flujo de la moléculas hacia la solución cesa y el sistema

alcanza el equilibrio cuando el menisco se ha elevado hasta una determinada altura, como se

puede observar en la figura 3.14. En estas condiciones de equilibrio, la solución se encuentra

L

Solución de azúcar

Agua

Membrana

Figura 3.12

Figura 3.13

Las moléculas de agua

atraviesan la membrana

Rebalse de la solución

Disolvente

puro

(Agua)

Figura 3.14

H = = gh

Las moléculas de

azúcar no pueden

atravesar la

membrana

Figura 3.12 - 3.14

Experimento de ósmosis. El agua pasa a través de la membrana a la solución de azúcar en el

compartimiento del sistema. El flujo de agua cesa cuando el líquido en el embudo ejerce hacia abajo

una presión suficiente (la presión osmótica).

Fig. 2.13

Page 22: Soluciones.y.propiedades

SOLUCIONES Y PROPIEDADES COLIGATIVAS

COMPENDIOS DE QUÍMICA GENERAL PARA ESTUDIANTES DE CIENCIAS E INGENIERÍA

78

bajo una presión hidrostática mayor que el disolvente puro. La altura del menisco

multiplicada por la densidad de la solución y la aceleración de la gravedad, dá la presión

adicional sobre la solución y ésta es la presión osmótica .

Por la medición experimental realizada en soluciones diluidas de concentración conocida, se

sabe que la relación entre la presión osmótica y la concentración está dada simplemente por:

= cRT (3.15)

Donde c es la concentración de la disolución en moles/litro, R es la constante universal de los

gases y T es la temperatura absoluta. La presión osmótica es proporcional a la temperatura

absoluta porque afecta el número de colisiones del disolvente con la membrana por unidad de

tiempo, también es proporcional a la concentración molar, ya que ésta afecta a la diferencia en

el número de moléculas del disolvente que chocan contra la membrana de cada lado.

Ejemplo 3.18.- Estime el peso molecular de una macromolécula biológica si se disuelve una

muestra de 0.194 gramos en 82.4 ml de benceno y se observa una presión osmótica de 14.20

torr a 25 °C.

Solución: De la ecuación (3.15) = cRT:

Considerando que el volumen de la solución es 82.4 ml, se puede determinar el número de

moles de la macromolécula biológica:

mol105.7680.0824mol

107 54

El peso molecular de dicha sustancia biológica es:

3.10.1 Ósmosis Inversa

l6 proceso de ósmosis inversa se ha aplicado al problema de la purificación del agua, en

particular el método se ha utilizado para desalar el agua del océano (esto es para eliminar

lar sales del agua de mar y obtener agua que se pueda beber o que sea industrialmente

utilizable). En la ósmosis normal, el disolvente fluye a través de una membrana de una

solución diluida a una solución mas concentrada, el proceso de ósmosis se puede detener.

Figura 3.15 Sistema de desalación que utiliza la ósmosis inversa.

Cuando el agua del océano se somete a una presión mayor que

su presión osmótica, el agua pura pasa a través de una serie de

membranas y dejan detrás una solución de sal más

concentrada. Si se aplica una presión aún mayor, el proceso

osmótico puede ser invertido, entonces el disolvente fluye de

la solución concentrada (que pueda ser agua del océano), a

través de una membrana, a una solución más diluida (que

pueda ser agua mas o menos pura). En la figura 3.15 se

presenta un sistema que utiliza ósmosis inversa para desalar el

agua del océano.

6

E

mol

g3363.38

5.768x10

0.194g

n

mM

5

mol107.64

298KmolK

mmHg62.4

mmHg 14.20

RT

πc 4

Page 23: Soluciones.y.propiedades

SOLUCIONES Y PROPIEDADES COLIGATIVAS

COMPENDIOS DE QUÍMICA GENERAL PARA ESTUDIANTES DE CIENCIAS E INGENIERÍA

79

3.11 Propiedades Coligativas de las Soluciones Ionicas

ara explicar las propiedades coligativas de las soluciones iónicas, debe aceptarse que la

concentración total de iones, es mas importante que la concentración de una sustancia

iónica. Por ejemplo, la depresión de congelación de una solución 0.100 m de cloruro de sodio

es casi el doble del de una solución 0.100 m de glucosa. Puede explicarse esto diciendo que el

cloruro de sodio se disuelve en agua para formar los iones Na+ y Cl

-. Cada unidad fórmula de

NaCl da dos partículas.

Para cada propiedad coligativa de las soluciones iónicas se pueden considerar tres valores: el

valor calculado Tm, suponiendo el compuesto formado por moléculas; el valor real, T,

bastante mayor, encontrado experimentalmente; y el valor ideal Ti, mayor todavía, que

puede también calcularse al suponer el compuesto formado por iones que se comportasen en

la disolución como si fueran partículas neutras..

3.11.1 Factor de van’t Hoff

La relación:

ΔTm

ΔTi (3.16)

donde i es mayor a la unidad, se conoce como factor de van’Hoff. Cuando se suponía que los

electrolitos estaban constituidos por moléculas se aceptó que una fracción de las mismas se

disociaba en iones y que se establecía un equilibrio entre las moléculas sin disociar y los iones

formados. La fracción de las moléculas ionizadas o disociadas se denomina grado de

disociación. Para los electrolitos fuertes se establece actualmente la relación:

gΔTi

ΔT (3.17)

donde g es menor que la unidad y que se conoce como coeficiente osmótico. Cuanto mas se

acerca a la unidad el valor de g mayor es el comportamiento ideal de los iones en la

disolución iónica.

Si una supuesta molécula del electrolito se disocia en iones, es evidente que:

Ti = Tm , y por tanto;

ig (3.18)

En el caso de los electrolitos débiles, si es el grado de ionización, y una molécula forma

realmente iones, 1 mol del electrolito dará lugar a moles y quedaran sin ionizar ( 1 - )

moles, por lo cual, en vez de un mol de compuesto tendremos:

[1+( - 1)]

moles de partículas, y puesto que cualquier propiedad coligativa es i veces mayor que el valor

teórico correspondiente al numero de moléculas o moles disueltos, tendremos

P

Page 24: Soluciones.y.propiedades

SOLUCIONES Y PROPIEDADES COLIGATIVAS

COMPENDIOS DE QUÍMICA GENERAL PARA ESTUDIANTES DE CIENCIAS E INGENIERÍA

80

i = 1 + ( - 1) (3.19) y

1

i (3.20)

Esta expresión se aplica también corrientemente a los electrolitos fuertes, aunque ya se ha

indicado que es incorrecto hablar en ellos de grado de disociación. Para no romper con esta

costumbre nos referiremos nosotros a un grado de disociación aparente para explicar el

comportamiento de los electrolitos fuertes.

El factor de van’t Hoff i para soluciones iónicas en las expresiones de las propiedades

coligativas se puede escribir:

Descenso de la presión de vapor

Ms

ms

Ms

mdMd

md

PP

i

(3.21)

Aumento del punto de ebullición

Te = iKem (3.22)

Descenso del punto de fusión

Tc = iKcm (3.23)

Presión osmótica

= icRT (3.24)

Ejemplo 3.19.- A 100 °C la presión de vapor de una disolución de 10.0 g de nitrato cálcico en

150 g de agua es de 746.8 mmHg. Hallar el grado de disociación aparente del nitrato cálcico

en esta disolución.

Solución: Para hallar el grado de disociación aparente (véase ecuación 3.15) del nitrato

cálcico (electrolito fuerte) debemos determinar el coeficiente de van’t Hoff a partir de la

ecuación (3.16) y el número de iones

Ca(NO3)2 = [ Ca++

+ 2NO3-] = 3

Así que, de la ecuación (3.16):

reemplazando datos:

efectuando operaciones con una máquina electrónica:

i 0.06088.33

8.330.982

Ms

ms

Ms

mdMd

md

PP

i

164

10

18

15018

150

7608.467

i

Page 25: Soluciones.y.propiedades

SOLUCIONES Y PROPIEDADES COLIGATIVAS

COMPENDIOS DE QUÍMICA GENERAL PARA ESTUDIANTES DE CIENCIAS E INGENIERÍA

81

i = 2.42

Aplicando la expresión (3.15)

Ejemplo 3.20.- Una disolución 0.065 molal de cloruro cálcico empieza a congelar a – 0.322

°C, hallar: a) el grado de disociación del cloruro cálcico y b) la concentración de los iones

cloruro y de los iones calcio en la disolución. Suponer la densidad de la disolución igual a la

unidad. Kc(H2O = 1.86 °C/mol)

Solución: a) Para hallar el grado de disociación haremos uso de la ecuación (2.15), sin

embargo es preciso evaluar i y .

El cloruro de calcio se disocia: CaCl2 = [ Ca

++ + 2Cl

- ] = 3

De la ecuación (3.18) Tc = iKcm: Despejamos i:

Por consiguiente

b) Para hallar la concentración molar de los iones, determinaremos en primera instancia la

concentración del cloruro de calcio: La masa total de la solución se puede calcular:

OH g 1000

CaCl g 7.215

CaCl mol 1

CaCl g 111

OH g 1000

CaCl mol 0.065

2

2

2

2

2

2

Es decir la masa de la solución es:

msolución = 1000 g

puesto que la densidad es 1.0 g/ml

El volumen de la solución es: V = 1000 ml = 1.000

La concentración molar de la solución iónica de CaCl2 es:

molar 0.065 1.000

CaCl de mol 0.065M 2

y las concentraciones de los iones determinamos a partir de : 0.065 0.83 = 0.0540

CaCl2 = Ca++

+ 2Cl-

1 mol 1 mol 2 mol

[ Ca++

]= 0.0540 molar, la relación es 1:1

71.013

142.2

1

1

i

2.660.0651.86

] 0.322)(0 [

m Kc

ΔTc

i

83.010013

166.2

Page 26: Soluciones.y.propiedades

SOLUCIONES Y PROPIEDADES COLIGATIVAS

COMPENDIOS DE QUÍMICA GENERAL PARA ESTUDIANTES DE CIENCIAS E INGENIERÍA

82

[ Cl- ] = 2 ( 0.0540) = 0.108 molar la relación es 1:2

PROBLEMAS RESUELTOS

3.1 Una solución de ácido sulfúrico tiene densidad de 1.84 g/ml y contiene 98% en masa de

H2SO4. ¿Qué volumen ocuparán 200 g de H2SO4 puro?

Solución: Se puede considerar los conceptos básicos de factores de conversión:

422 SOHOH3 VVcm91.110 (1)

El volumen de agua es:

En consecuencia, considerando la ecuación (1), el volumen de H2SO4 puro es:

423

SOH SOHcm82.106082.491.110V42

3.2 Calcule la molalidad, fracciones molares de soluto y disolvente, y la molaridad de las

siguientes soluciones: a) una solución acuosa de H2SO4 que es 50 % en masa y tiene una

densidad de 1.4 g/ml, b) una solución acuosa de sacarosa, C12H22O11, que es 19 por ciento

sacarosa en masa, y tiene una densidad de 1.08 g/ml, y c) una solución compuesta de 24.4 g

de NaOH y 97.6 de H2O con un volumen de 100 ml.

Solución: a) La molalidad de la solución de H2SO4 se puede determinar a partir de la

composición centesimal, vale decir:

molal 10.2mSOH g 98

SOH mol 1

OH kg 1

OH g 1000

OH g 50

SOH g 50m

42

42

2

2

2

42

La fracción molar se determina calculando previamente los moles del soluto y disolvente:

OH mol 2.78OH g 18

OH mol 1OH g 50

SOH mol 0.510SOH g 98

SOH mol 1SOH g 50

2

2

22

42

42

4242

La fracción molar será entonces :

845.0155.01X

155.0290.3

510.0X

OH

SOH

2

42

La molaridad se calcula considerando la densidad de la solución

solución de cm 110.91solución de g 1.84

solución de cm 1

SOH g 98

solución de g 100SOH g 200 3

3

4242

OH cm 4.082OH g 1

OH cm 1

solución de g 100

OH de g 2

SOH g 98

solución de g 100SOH g 200 2

3

2

23

2

4242

Page 27: Soluciones.y.propiedades

SOLUCIONES Y PROPIEDADES COLIGATIVAS

COMPENDIOS DE QUÍMICA GENERAL PARA ESTUDIANTES DE CIENCIAS E INGENIERÍA

83

molar 7.14Solución 1

mlSol. 1000

SOH g 98

SOH mol 1

Solución g 100

SOH g 50

Solución ml

Solución g1.4M

42

4242

b) La molalidad de la solución de C12H22O11, se puede determinar a partir de la composición

centesimal, vale decir:

molal 0.6859OHC g 342

OHC mol 1

OH kg 1

OH g 1000

OH g 81

OHC g 19m

112221

112221

2

2

2

112221

La fracción molar se determina calculando previamente los moles del soluto y disolvente:

OH mol 4.50OH g 18

OH mol 1OH g 81

OHC mol 0.0555nOHC g 342

OHC mol 1OHC g 19n

2

2

22

112212OHC

112212

112212112212OHC 112212112212

La fracción molar será entonces :

988.0012.01X

012.05555.4

0555.0X

OH

OHC

2

112212

La molaridad se calcula considerando la densidad de la solución

molar 0.6Solución 1

Solución ml 1000

11O22H12C g 342

11O22H12C mol 1

Solución g 100

11O22H1219gC

Solución ml

Solución g1.08M

3.3 Una solución acuosa de cloruro de sodio que contiene 8.00 g de NaCl por 100 g de

solución posee una densidad de 1.054 g/ml a la temperatura de 25ºC. Determine: a) la

concentración molar y b) la concentración molal de la solución.

Solución:

a) 8 g NaCl/100 g sol. Molaridad =solución de litro 1

soluto de moles Nº

= 1.054 g/ml

moles de soluto :

n = NaCl mol 0.137g/mol 58.5

g 8

M

m

-Cálculo del volumen en litros de solución:

ml 94.88g 1.054

solución de ml 1 g 100V

V= 0.0949 litros

Molaridad :

Page 28: Soluciones.y.propiedades

SOLUCIONES Y PROPIEDADES COLIGATIVAS

COMPENDIOS DE QUÍMICA GENERAL PARA ESTUDIANTES DE CIENCIAS E INGENIERÍA

84

molar 1.44 0.0949

NaCl mol 0.137M

b) Molalidad = solvente de kg

soluto de moles de Nº

Cálculo de la masa en kilogramos de solvente

Masa del solvente = masa (solución) - masa (soluto) = 100 g - 8 g = 92 g de solvente

Kg de solvente = 92g = 0.092 Kg

Molalidad = molal 1.49solvente de kg 0.092

soluto de mol 0.137

3.4 Se prepara 1.5 litros de una solución de H2O y C2H5OH, cuya densidad relativa resulta

0.947 en una proporción volumétrica de 3:1 respectivamente. Determinar: a) La densidad del

alcohol etílico. b) La fracción molar del alcohol etílico. c) La molaridad. d) La molalidad.

Solución: Datos: Vsolución = 1.5

solución = 0.947 kg/

%VH2O = 75%

%VC2H5OH = 25%

a) VH2O = 1.5 0.75 = 1.125

VC2H5OH = 1.5 0.25 = 0.375

sol

solsol

V

kg 1.4205m

1.5kg

0.947Vρm

sol

solsolsol

En consecuencia la masa del alcohol etílico es: mC2H5OH = msol – mH2O

mC2H5OH =1.4305 – 1.125 = 0.2955 kg

mC2H5OH = 295.5 g

La densidad del alcohol etílico es:

0.788g/ml375ml

295.5g

V

b) T

OHHC

OHHCn

nχ 52

52

mol 6.424

mol

g 46

g 295.5n OHHC 52

mol 62.5

mol

g 18

g 1125n OH2

nT = 6.424+62.5= 68.924 mol

093.0

924.68

424.652

mol

molOHHC

Page 29: Soluciones.y.propiedades

SOLUCIONES Y PROPIEDADES COLIGATIVAS

COMPENDIOS DE QUÍMICA GENERAL PARA ESTUDIANTES DE CIENCIAS E INGENIERÍA

85

molar 4.28solución 1.5

OHmolHC 6.424nM 52

solución

soluto

molal 5.71OH kg 1.125

OHHmolC 6.424

kg

nm

2

52

edilsolvent

soluto

c)

d)

3.5 ( 20 puntos, 20 min) La densidad relativa de una disolución acuosa de cloruro de potasio

que contiene 24.6 g KCl es de 1.131 a 21 ºC, mientras que la densidad relativa del cloruro de

potasio sólido, a la misma temperatura es 1.984. Calcular: a) la molaridad, b) la normalidad,

c) la molalidad, d) la fracción molar, y e) el tanto por ciento en volumen. (K = 39; Cl = 35.5)

Solución: Datos: Solución= 1.131 g/ml; KCl= 1.984 g/ml; H2O= 1.0 g/ml;

mKCl = 24.6 g mH2O = x

OHKCl

OHKCl

total

totalsolución

VV

mm

V

m

2

2

(1)

mH2O = VH2O (2)

mlgm

VKCl

KClKCl 40.12

984.1

6.24

(3)

Reemplazando en (1)

OH

OH

m

mg

2

2

4.12

6.24131.1

mH2O = 80.76 g

VH2O = 80.76 ml

Volumen total: Vsolución = 12.4 ml + 80.76 ml = 93.16 ml

a) la molaridad es:

molargKCl

molKClml

ml

gKCl54.3

5.74

1

1

1000

16.93

6.24

b) La normalidad es:

normalmolKCl

gKCleq

gKCl

molKClml

ml

gKCl54.3

1

1

5.74

1

1

1000

16.93

6.24

c) La molalidad:

molalgKCl

molKCl

OKgH

OgH

OgH

gKCl09.4

5.74

1

1

1000

76.80

6.24

2

2

2

d) La fracción molar:

molKClgKCl

molKClgKCl 33.0

5.74

16.24 OmolH

OgH

OmolHOgH 2

2

22 49.4

18

176.80

068.082.4

33.0KClX 932.0

82.4

49.42

OHX

e) El % en volumen: %31.13%10016.93

4.12%

ml

mlV KCl

Page 30: Soluciones.y.propiedades

SOLUCIONES Y PROPIEDADES COLIGATIVAS

COMPENDIOS DE QUÍMICA GENERAL PARA ESTUDIANTES DE CIENCIAS E INGENIERÍA

86

3.6.- Determinar: a) la concentración molal, b) la fracción molar y la molaridad

de una solución de ácido sulfúrico del 50% en masa y cuya densidad es 1.4 g/ml.

Solución: a) La molalidad de la solución de H2SO4 se puede determinar a partir de la

composición centesimal, vale decir:

molal 10.2SOH g 98

SOH mol 1

OH kg 1

OH g 1000

OH g 50

SOH g 50m

42

42

2

2

2

42

La fracción molar se determina calculando previamente los moles del soluto y disolvente:

OH mol 2.78OH g 18

OH mol 1OH g 50

SOH mol 0.510SOH g 98

SOH mol 1SOH g 50

2

2

22

42

42

4242

La fracción molar será entonces :

845.0155.01X

155.0290.3

510.0X

OH

SOH

2

42

La molaridad se calcula considerando la densidad de la solución

3.7 A 100 ml de una disolución de ácido sulfúrico del 96% en masa y de densidad relativa

1.84, se añadieron 400 ml de agua, obteniéndose una solución de peso específico relativo

igual a 1.22. En la solución resultante determinar:

a) la concentración en tanto por ciento en masa.

b) La concentración molar

c) La concentración molal

d) La normalidad

e) La fracción molar

Solución: El lector debe hacer énfasis en esquemas de manera que le permitan ver

objetivamente el problema:

A 100 ml de una solución de ácido sulfúrico se añade 400 ml de agua, a esta operación se

denomina dilución, que obviamente modificará la concentración y la densidad de la solución.

Para resolver el problema calcularemos el volumen de agua de la solución original para luego

sumar a los 400 ml de agua que se añadieron, a partir de este dato la masa de agua de modo

V = 100 ml

96%H2SO4

= 1.84

V = ¿?

%H2SO4 =¿?

= 1.22

H2O

molar 7.14Solución 1

Solución ml 1000

SOH g 98

SOH mol 1

Solución g 100

SOH g 50

Solución ml

Solución g1.4M

42

4242

Page 31: Soluciones.y.propiedades

SOLUCIONES Y PROPIEDADES COLIGATIVAS

COMPENDIOS DE QUÍMICA GENERAL PARA ESTUDIANTES DE CIENCIAS E INGENIERÍA

87

que sumados a la masa de ácido sulfúrico puro y considerando la densidad de la solución

resultante se podrá determinar el volumen de la solución resultante.

Determinación de la masa de H2O y H2SO4 de la solución original

la masa de agua en la solución resultante será:

mH2O = (7.36 + 400) g = 407.36 g H2O

De acuerdo a algunas estrategias recomendadas en el presente texto es fundamental conocer la

concentración de la solución resultante en tanto por ciento:

a) El tanto por ciento del ácido sulfúrico se determina a partir de:

m H2SO4 = 176.64 g

m H2O = 407.36 g

La masa total es: (176.64 + 407.36) g = 584 g

43.36%100%407.36

176.64SO%H 42

b) La concentración molar se determina conociendo la densidad de la solución y la

concentración en tanto por ciento en masa de ácido sulfúrico:

molar 5.398M

SOH g 98

SOH mol 1

solución g 100

SOH g 43.36

solución de 1

solución ml 1000

solución ml

solución g1.22M

42

4242

c) Determinación de la molalidad:

Sólo se considera la concentración en tanto por ciento en masa:

Masa de H2SO4 = 43.36 g

Masa de H2O = (100 - 43.36) g = 56.64 g

En consecuencia:

molal 7.811SOH g 98

SOH mol 1

OH kg 1

OH g 1000

OH g 56.64

SOH g 43.36molalidad

42

42

2

2

2

42

d) Determinación de la normalidad:

Se puede calcular a partir de la molaridad considerando que en esta solución:

OH g 7.36g 176.64g 184m

SOH g 176.64solución de 100g

SOH g 96solución de g 184

solución de g 184sol. de ml 1

sol. de g 1.84sol. de ml 100

2agua

4242

Page 32: Soluciones.y.propiedades

SOLUCIONES Y PROPIEDADES COLIGATIVAS

COMPENDIOS DE QUÍMICA GENERAL PARA ESTUDIANTES DE CIENCIAS E INGENIERÍA

88

4242

4242

SOH mol 1SOH geq 2

2

SOH mol 1SOH geq 1

Por tanto:

d) Determinación de la fracción molar:

Para calcular la fracción molar requerimos los moles del soluto y del disolvente:

mol 22.63OH g 18

OH mol 1OH g 36.074OnH

mol 1.802SOH g 98

SOH mol 1SOH g 176.64SOnH

2

222

42

424242

Las fracciones molares son:

0.9324.43

22.63 χ

0.0724.43

1.80 χ

OH

SOH

2

42

3.8 Una instalación de cámaras de plomo suministra diariamente 12 m3 de ácido de cámara de

densidad relativa 1.49 y de una concentración del 60% en masa. Si se desea preparar un ácido

concentrado del 98%

a) ¿Qué cantidad de agua se debe evaporar diariamente?

b) Calcular la normalidad, molaridad y la molalidad del ácido de cámara.

Solución: El ácido de cámara del que se habla no es más que el ácido sulfúrico.

a) Efectuaremos un balance másico sin reacción química puesto que se trata de un proceso

físico, donde la masa de la solución mA de ácido de cámara es:

t17.88m

t1.49m 12m

3

3A

7Asumiremos como base de cálculo 17.88 t por día.

7 5 Fuente: Viceministerio de Industria y comercio interno. De acuerdo al Sistema Internacional de unidades el

símbolo de tonelada métrica es t

Ácido de cámara

(60%) H2SO4

Agua evaporada

(0%) H2SO4 Ácido resultante

( 98%) H2SO4

A

Q

B

Q C

Q

N 10.796SOH mol 1

SOH geq 2

solución 1

SOH moles5.398

42

4242

Page 33: Soluciones.y.propiedades

SOLUCIONES Y PROPIEDADES COLIGATIVAS

COMPENDIOS DE QUÍMICA GENERAL PARA ESTUDIANTES DE CIENCIAS E INGENIERÍA

89

mA = mB + mC (1)

amA = bmB + cmC (2)

En la ecuación (2) b = 0, por tanto la masa la masa del ácido resultante mc es:

t10.94798

t17.8860

c

amm a

c

De la ecuación (1) despejamos la masa de agua:

mB = mA - mC

mB = 17.880 - 10.947 = 6.933 t H2O

Puesto que la densidad del agua es 1 t/m3

El volumen de agua que se evapora diariamente es de 6.933 m3

b) La concentración molar del ácido de cámara se puede determinar a partir de su densidad y

tanto por ciento en peso.

La normalidad se puede calcular a partir de la molaridad considerando que en esta solución:

4242

4242

SOH mol 1SOH geq 2

2

SOH mol 1SOH geq 1

Por tanto:

Para determinar la molalidad sólo se considera la concentración en tanto por ciento en masa:

masa de H2SO4 = 60.0 g

masa de H2O = (100 – 60.0) g = 40.0 g

En consecuencia:

molal 31.15SOH g 98

SOH mol 1

OH kg 1

OH g 1000

OH g 40.0

SOH g 60.0molalidad

42

42

2

2

2

42

3.9 Una disolución de hidróxido ferroso a 25 ºC de temperatura tiene una solubilidad de 1.78

g/100g H2O. Determinar: a) la molaridad, b) la normalidad, c) la molalidad y la fracción

molar del soluto suponiendo solución ideal.

Solución: a) Cuando la solución es ideal la densidad de la solución tiende a ser la densidad del

agua, sin embargo la masa total de la solución se debe considerar:

molar 12.9SOH g 98

SOH mol 1

solución g 100

SOH g 60.00

solución de 1

solución ml 1000

solución ml

solución g1.49M

42

4242

N 18.24SOH mol 1

SOH geq 2

solución 1

SOH moles9.12

42

4242

Page 34: Soluciones.y.propiedades

SOLUCIONES Y PROPIEDADES COLIGATIVAS

COMPENDIOS DE QUÍMICA GENERAL PARA ESTUDIANTES DE CIENCIAS E INGENIERÍA

90

(100 g + 1.78g) = 101.78 g

molar 0.194solución de litro 1

solucióncm 000 1

solución cm 1

solución g 1

Fe(OH) g 90

Fe(OH) mol 1

solución g 101.78

Fe(OH) g 1.78 3

32

22

b) Para determinar la normalidad de la solución, hallamos la relación de moles y equivalentes

gramo del soluto:

22

22

Fe(OH) mol 1Fe(OH) geq 2

2

Fe(OH) mol 1Fe(OH) geq 1

N 0.388Fe(OH) mol 1

Fe(OH) geq 2

solución de 1

(FeOH) mol0.194

2

22

c) A partir de la solubilidad es posible calcular la molalidad:

molal 0.198agua de kg 1

agua g 000 1

Fe(OH) g 90

Fe(OH) mol 1

agua de g 100

Fe(OH) g 1.78

2

22

a) Las fracciones molares serán:

mol 5.56g/mol 18

OH g 100n

mol 0.0198g/mol 90

Fe(OH) g 1.78n

2OH

2Fe(OH)

2

2

0035.05.5798

0.01982Fe(OH)

3.10 Se tiene una solución de dicromáto de potasio al 1%, considerando solución ideal,

calcular el volumen de esta solución que se necesita para preparar 250 cm3 de solución 0.1

normal de dicromato al actuar como oxidante.

Solución: Para resolver éste problema se debe tomar en cuenta que el dicromáto actúa como

oxidante, esto implica tener cuidado en el cálculo del equivalente gramo ya que se trata de una

reacción de oxidación y reducción.

El dicromáto de potasio al entrar en contacto con el agua se disocia según:

K2Cr2O7= = 2 K

+ + Cr2O7

=

En solución el dicromáto se reduce de acuerdo a:

6e- + 14H

+ + Cr2O7

= 2Cr

+3 + 7H2O

0

En consecuencia el equivalente gramo del dicromáto de potasio es:

722722

722722

OCrK mol 1OCrK geq 6

6

OCrK mol 1OCrK geq

Page 35: Soluciones.y.propiedades

SOLUCIONES Y PROPIEDADES COLIGATIVAS

COMPENDIOS DE QUÍMICA GENERAL PARA ESTUDIANTES DE CIENCIAS E INGENIERÍA

91

N 0.204solución de litro 1

solución de cm 1000

solución de cm 1

solución de g 1

OCrK mol 1

OCrgKeq 6

OCrK g 294

OCrK mol 1

solución de g 100

OCrK g 1

3

3722

722

722

722722

Al igualar los equivalentes de las dos soluciones se tiene:

2211 VNVN

33

2 cm 122.55N 0.204

N 0.1cm 250V

3.11 Se queman 80 litros de fosfamina medidos a 18 °C y 756 mmHg. El ácido fosfórico

formado se disuelve en agua y se obtiene un litro de disolución. Calcular la normalidad de la

disolución ácida.

Solución: El proceso de formación de una disolución de H3PO4 viene a partir de la siguiente

ecuación química:

PH3 + 2O2 H3PO4

A partir de esta ecuación química se calcula estequiométricamente la cantidad de sustancia de

H3PO4

Considerando la ecuación de estado:

PV = nRT se tiene:

43

3

433

3

POH mol 3.33PH mol 1

POH mol 1NH mol 3.33

NH mol 3.33

K 291molK

mmHg62.4

80mmHg 756

RT

PVn

La relación entre moles del ácido y los eq-g se obtiene:

4343

4343

POH mol 1POH g3eq

3

POH mol 1POH g1eq

La normalidad de la solución ácida será:

N 10POH mol 1

POH gEq 3

solución de litro 1

POH mol 3.33N

43

4343

3.12 Calcular el número de gramos de sulfato ferroso que se oxidarán en una disolución de

esta sal acidificada con ácido sulfúrico, con 24 ml de permanganato de potasio, en solución

0.25 N, considerando como agente oxidante para esta reacción.

Page 36: Soluciones.y.propiedades

SOLUCIONES Y PROPIEDADES COLIGATIVAS

COMPENDIOS DE QUÍMICA GENERAL PARA ESTUDIANTES DE CIENCIAS E INGENIERÍA

92

Solución: Formulamos e igualamos la ecuación química por el método ión - electrón

FeSO4 + H2SO4 + KMnO4 Fe2(SO4)3 + MnO + K2SO4 + H2O

Las semirreacciones son:

2 Fe+2

2 Fe+3

+ 2 e-

(1)

5 e- + 8H

+ + MnO4

- Mn

+2 + 4H2O (2)

multiplicando la primera ecuación por 5 y la ecuación (2) por 5:

10 Fe+2

+ 16H+ + 2 MnO4

-

10 Fe

+3 + 2 Mn

+2 + 8H2O

La ecuación igualada es

10FeSO4 + 8 H2SO4 + 2 KMnO4 5 Fe2(SO4)3 + 2 MnSO4 + K2SO4 + 8 H2O

Para la determinación del eq – g de KMnO4, debemos considerar los electrones ganados

porque se trata de una reacción de oxidación – reducción.

KMnO4 mol 1KMnO4 geq 5

5

KMnO4 mol 1KMnO4 geq 1

La masa de sulfato ferroso que se oxida es:

4

4

4

4

4

4

444

FeSO g 0.912FeSO mol 1

FeSO g 152

KMnO mol 2

FeSO mol 10

KMnO geq 5

KMnO mol 1

solución ml 1000

KMnO gEq 0.25KMnO ml 24

Propiedades coligativas:

3.13 A 25 ºC la presión de vapor saturado del agua constituye 23.76 mmHg. Hallar a la

misma temperatura, la presión del vapor saturado sobre una disolución acuosa al 5% de

carbamida , CO(NH2)2.

Solución: Aplicaremos la ley de Rault:

P = Xd.Pº

La fracción molar del disolvente se puede evaluar a partir de la composición que nos dan en el

problema, vale decir:

5 g de carbamida y 95 g de agua

Page 37: Soluciones.y.propiedades

SOLUCIONES Y PROPIEDADES COLIGATIVAS

COMPENDIOS DE QUÍMICA GENERAL PARA ESTUDIANTES DE CIENCIAS E INGENIERÍA

93

985.05.363

5.28

OH mol 5.28OH g 18

OH mol 1OgH 95

)CO(NH mol 0,083

g 60

)CO(NH mol 1)CO(NH g 5

OH

2

2

22

22

2222

2

P = 0.985 23.76 mmHg

3.14 La solución saturada de fenol en agua tiene a 21 ºC la presión de vapor de 18.3 mmHg,

mientras que la del agua pura es 18.65 mmHg. a) Determinar la solubilidad del fenol en agua ,

suponiendo la idealidad. Exprese también el resultado como b) molaridad, c) molalidad y d)

tanto por ciento en masa.

Solución: Datos:

P = 18.3 mmHg P° = 18.65

a) solubilidad = ? b) M = ? c) m = ? d) % masa = ?

a) La solubilidad se define:

disolvente de g 100

soluto gdsolubilida

Aplicaremos la ley de Rault:

De la ecuación (3.5): P = P°:

Ms

ms

Ms

mdMd

md

PP

reemplazando datos:

9418

10018

100

65.183.18sm

efectuando operaciones con una máquina electrónica:

OHHC g 10.12m

1077.094

m

56s

s

Por tanto la solubilidad es:

agua de g 100

soluto de g 10.12dsolubilida

b) molaridad

Para soluciones ideales se considera la densidad 1 g/ml La masa total de la solución es:

(100 + 10.12) g = 110.12 g

Page 38: Soluciones.y.propiedades

SOLUCIONES Y PROPIEDADES COLIGATIVAS

COMPENDIOS DE QUÍMICA GENERAL PARA ESTUDIANTES DE CIENCIAS E INGENIERÍA

94 V = 110.12 g / 1 g/ml = 110.12 ml = 0.11 litros

Los moles de fenol: 10.12 g / 94 = 0.108 mol

molar 0.98solución litros 0.11

OHHC mol 0108M 56

c) molalidad

molal 1.08OH kg 0.10

OHHC mol 0.108m

2

56

d) % masa:

OHH9.19%C100%110.12

10.12OHH%C 5656

3.15 Cuando se disuelven 60 g de una mezcla de glucosa (C6H12O6) y sacarosa (C12H22O11)

en 200 g de agua, se registra una presión de vapor en la disolución de 23.28 mmHg a 25 ºC.

La presión de vapor del agua pura a dicha temperatura es 23.76 mmHg. Determinar la

composición porcentual en masa en la mezcla de glucosa y sacarosa.

Solución Datos:

60 g de mezcla de C6H12O6 y C12H22O11

sean :

C6H12O6 = x y C12H22O11 = y en 200 g de H2O.

P = 23.28 mmHg Pº = 23.76 mmHg

Aplicando la ley de Rault: P = PºXdisolvente;

despejando Xd:

98.076.23

28.23

º

mmHg

mmHg

P

PX d

por definición de fracción molar:

solutoOH

OH

disolventenn

nX

2

2

0.98

M

60

18

20018

200

M

m

M

m

M

m

X

soluto

soluto

OH

OH

OH

OH

disolvente

2

2

2

2

reemplazando datos: mol

gM 63.264

Considerando la ecuación: nx + ny = nT

63.264

60

342180

yx (1)

60 yx (2)

Page 39: Soluciones.y.propiedades

SOLUCIONES Y PROPIEDADES COLIGATIVAS

COMPENDIOS DE QUÍMICA GENERAL PARA ESTUDIANTES DE CIENCIAS E INGENIERÍA

95

Resolviendo el sistema de ecuaciones se determina:

x = 19.6 g %C6H12O6 = 32.67%

y = 40.4 g %C12H22O11 = 67.33%

3.16 Calcular la cantidad de C2H6O2 que se debe añadir a 1 kg de etanol para reducir su

presión de vapor en 9.5 mmHg a 35 ºC. La presión de vapor del etanol puro a esta temperatura

es 100 mmHg.

Solución: La presión de vapor de la solución es:

(100 - 9.5) mmHg = 90.5 mmHg

Considerando la ley de Rault: P = PºXdisolvente;

905.0mmHg 100

mmHg 90.5

PXd

por definición de fracción molar:

solutoOH

OH

disolventenn

nX

2

2

0.905

62

m

46

100046

1000

M

m

M

m

M

m

X

soluto

soluto

OHHC

OHHC

OHHC

OHHC

disolvente

52

52

52

52

282.262

sm

ms = 141.49 g C2H6O2

3.17 A 30 ºC, la presión de vapor del éter dietílico es de 646 mmHg y de la acetona pura de

283 mmHg. Calcule la composición molar de una mezcla cuya presión sea de 460 mmHg,

suponiendo la idealidad.

Solución: Puesto que ambas sustancias son volátiles (A = éter dietílico; B = acetona) se

cumple que:

Ptotal = PA + PB = XA.PAº + XB.PBº

460 = A646 + B283 (1)

A + B = 1 (2)

A = 1 - B (3)

(3) reemplazando en (1): 460 = 646 (1 - B ) + 383 B

Despejando la fracción molar de la acetona

B = 0.51; A = 0.49

Page 40: Soluciones.y.propiedades

SOLUCIONES Y PROPIEDADES COLIGATIVAS

COMPENDIOS DE QUÍMICA GENERAL PARA ESTUDIANTES DE CIENCIAS E INGENIERÍA

96

La composición molar es

%nacetona = 0.51100 = 51.0%

%néter = 0.49100 = 49.0%

3.18 En un compuesto orgánico recientemente sintetizado, al realizar el análisis químico se ha

encontrado que contiene 63.2% de carbono, 8.8% de hidrógeno y el resto oxígeno. Una

disolución de 0.0702 gramos de este compuesto en 0.804 g de alcanfor deprime el punto de

solidificación en 15.3 °C. ¿Cuál es la fórmula molecular del compuesto desconocido? La

constante crioscópica del alcanfor es igual a 40.

Solución: Determinación de la fórmula empírica:

1.7516

18:O

8.81

8.8:H

5.26612

63.2:C

11.75

1.75:O

51.75

8.8:H

31.75

5.266:C

La fórmula empírica es: C3H5O (M = 57)

Para determinar la fórmula molecular debemos calcular el peso molecular de la sustancia a

partir de la expresión del descenso crioscópico.

Tc = Kcm

m = 15.3 °C / 40 °C/mol = 0.3825

El peso molecular calculamos a partir de la expresión (3.8)

MA

1000am

27.2283825.0804.0

10000702.0

mA

1000aM

En la expresión: (C3H5O)n

457

27.228n

La fórmula molecular es: C12H20O4

3.19 Hallar la concentración en gramos por litro de una solución de glucosa C6H12O6 que a 25

°C es isosmótica con una disolución de úrea CO(NH2)2 a 12 °C la cual contiene 3 g de úrea en

un volumen de 150 cm3.

Solución El término isosmótico implica la misma presión osmótica pero a diferentes

temperaturas, así que se puede escribir la expresión de presión osmótica para ambas

sustancias e igualarlas.

= cRT

Page 41: Soluciones.y.propiedades

SOLUCIONES Y PROPIEDADES COLIGATIVAS

COMPENDIOS DE QUÍMICA GENERAL PARA ESTUDIANTES DE CIENCIAS E INGENIERÍA

97

Para la solución de glucosa se tiene:

1 = c1RT1 (1)

Para la solución de úrea: 2 = c2RT2 (2)

T1 = 25 + 273 = 298 K

T2 = 12 + 273 = 285 K

La concentración C2, de la úrea es:

molar 0.333solución litro 1

solución ml 1000

úrea 60g

úrea de mol 1

solución ml 150

úrea g 3C2

Igualando las ecuaciones (1) y (2)

c1RT1 = c2RT2

Despejando C1:

g/ 57.24OHC 1mol

OHC g 180

solución de litro 1

OHC moles0.318

0.318molar0.333molar298

285c

T

Tc

6126

61266126

2

1

21

3.20 El bromuro de etileno, C2H4Br2, y el 1,2 dibromopropano C3H6Br2, forman una serie de

disoluciones ideales en todas sus concentraciones. A 85 ºC, la presión de estos dos líquidos

puros es 173 y 127 torr respectivamente. a) Si se disuelven 10 g de bromuro de etileno en 80 g

de 1,2 dibromopropano, calcúlese la presión de cada componente y la presión total de la

solución a 85 ºC. b) Calcúlese la fracción molar del bromuro de etileno en el vapor en el

equilibrio con la solución anterior. c) Cuál será la fracción molar del bromuro de etileno en

una solución a 85 ºC en equilibrio con una mezcla de 50:50 moles en el vapor?

Solución: Para calcular la presión de cada componente se debe considerar la ley de Rault:

PC2H4Br2 = C2H4Br2 P° C2H4Br2

PC2H4Br2 = C3H6Br2 P° C3H6Br2

P° C2H4Br2 = 173 mmHg

P° C3H6Br2 = 127 mmHg

Las fracciones molares son:

242263

263263

242242

242242

BrHC mol 0.396BrHC g 201.84

BrHC mol 1BrHC g 80

BrHC mol 0.053BrHC g 187.84

BrHC mol 1BrHC g 10

0.8820.1181χ

0.1180.449

0.053χ

263

242

BrHC

BrHC

Las presiones de vapor parciales son:

PC2H4Br2 = 0.118 173 mmHg = 20.414 mmHg

Page 42: Soluciones.y.propiedades

SOLUCIONES Y PROPIEDADES COLIGATIVAS

COMPENDIOS DE QUÍMICA GENERAL PARA ESTUDIANTES DE CIENCIAS E INGENIERÍA

98 PC3H6Br2 = 0.882 127 mmHg =112.014 mmHg

La presión de vapor de la solución es:

Ptotal = PC2H4Br2 + PC3H6Br2 = (20.414 + 112.014) mmHg

Ptotal = 132.428 mmHg

Puesto que se trata de soluciones ideales, su comportamiento puede mostrarse en un diagrama

fracción molar vs presión de vapor de ambas soluciones:

Del diagrama realizada a escala se puede dar una

interpretación del comportamiento de estas

soluciones.

Por ejemplo se puede dar eventualmente la presión

total de la solución ideal cuando las fracciones

molares de ambas sustancias son iguales, es decir

0.5.

Trace una línea en la fracción molar 0.5 y lea la

intersección en presión de vapor esto da

aproximadamente 150 mmHg. en la práctica resulta

muy útil el uso de estos diagramas.

c) Para determinar las fracciones molares de estas

sustancias en fase vapor, debemos recordar un

concepto muy importante de las fracciones molares en función de las presiones parciales.

total

AA

P

Entonces la fracción molar del bromuro de etileno en la fase vapor es:

0.154132.428

20.414χ

242 BrHC

d) La fracción molar del bromuro de etileno en la fase líquida cuando las fracciones molares

en fase gaseosa son iguales será:

Sean: A = C2H4Br2 y B = C3H6Br2

En fase gaseosa se cumple: T

AA

P

PX '

y T

BB

P

PX '

(1)

Además: AAA XPP BBB XPP (2)

Donde X’A y X’B son las fracciones molares en fase gaseosa y XA y XB, son las fracciones

molares en solución

Despejando PA y PB de (1)

0.12 0.5 1.0

Bromuro de etileno

0.88 0.5

Dibromo propano

127

173

132

Pre

sió

n d

e v

apo

r [m

mH

g]

150

Page 43: Soluciones.y.propiedades

SOLUCIONES Y PROPIEDADES COLIGATIVAS

COMPENDIOS DE QUÍMICA GENERAL PARA ESTUDIANTES DE CIENCIAS E INGENIERÍA

99

'ATA XPP '

BTB XPP (3)

Igualando ahora las ecuaciones (2) y (3) 'ATAA XPXP 'BTBB XPXP

Dividiendo estas ecuaciones y tomando en cuenta que X’A = X’B = 0.5

BBAA XPXP

734.0173

127

torr

torr

P

P

X

X

A

B

B

A

Considerando además que:

XA + XB = 1

Y resolviendo el sistema de ecuaciones, encontramos que las fracciones molares en fase líquida son:

XA = 0.42

XB = 0.58

3.21 El benceno congela a 5.5 °C y hierve a 80.2 °C. Los calores latentes de fusión y

ebullición del benceno son, respectivamente, 30.3 y 94.2 cal/g. Calcular las constantes

molales de los puntos de a) congelación y de b) ebullición del benceno. Los valores

experimentales so 5.12 y 2.67 °C/molal, respectivamente

Solución: Las constantes molales dependen del calor latente de ebullición y fusión

respectivamente, y aproximadamente se puede calcular considerando la siguiente ecuación:

a)

K/mol 09.5

cal/g 3.03g 1000

278.5KmolK

cal1.987

1000Δ00

RTKc

22c

b)

K/mol 2.63

cal/g 94.2g 1000

353.2KmolK

cal1.987

1000Δ00

RTKe

22e

3.22 En un aparato para helados de tipo casero se abate el punto de congelación de un baño

de agua que rodea el helado disolviendo NaCl para obtener una solución salina. Se observa

que una solución salina al 15% se congela a – 10.880 °C. ¿Cuál es el factor de van’t Hoff, i,

para esta solución?

Solución: Datos

Tf(agua) = 0 °C Tf(solución) = -10.88 °C

Kc = 1.86 °C/m

Page 44: Soluciones.y.propiedades

SOLUCIONES Y PROPIEDADES COLIGATIVAS

COMPENDIOS DE QUÍMICA GENERAL PARA ESTUDIANTES DE CIENCIAS E INGENIERÍA

100

molal 3.02mOH kg 1

OH g 1000

NaCl g 58.5

NaCl mol 1

OH g 85

NaCl g 15m

2

2

2

Consideremos la ecuación Tc = iKcm y despejando i, se tiene:

m Kc

ΔTc

i

94.1

02.386.1

)88.10(0

mm

C

CCi

3.23 El cloruro de cesio se disuelve en agua según la siguiente reacción:

CsCl Cs+

+ Cl-

Una solución 0.121 m de CsCl se congela a – 0.403. Calcule i y la disociación porcentual de

CsCl en esta disolución.

Solución:

Considerando la ecuación Tc = iKcm y despejando i, se tiene:

m Kc

ΔTc

i

donde: Tc = 0 °C – (- 0.403 °C) = 0.403 °C

El factor de Van’t Hoff será:

1.7906

0.121mm

C1.86

C0.403

i

La disociación porcentual se calcula considerando la ecuación:

1

i

los iones formados, como se observa en la ecuación de disociación es: = 2, reemplazando en

la anterior expresión:

0.790612

11.7906α

La disociación porcentual es: = 0.7906 100% = 79.06%

3.24 El compuesto complejo, K3[Fe(CN)6], se disuelve en agua según la siguiente reacción:

K3[Fe(CN)6] 3K+ + [Fe(CN)6

-3]

Una solución 0.126 m de K3[Fe(CN)6] se congela a – 0.649 °C. Calcule el factor de Vant’

Hoff y la disociación porcentual de K3[Fe(CN)6] en esta solución.

Page 45: Soluciones.y.propiedades

SOLUCIONES Y PROPIEDADES COLIGATIVAS

COMPENDIOS DE QUÍMICA GENERAL PARA ESTUDIANTES DE CIENCIAS E INGENIERÍA

101

Solución: Considerando la ecuación Tc = iKcm y despejando i, se tiene:

m Kc

ΔTc

i

donde Tc = 0 °C – (- 0.649 °C) = 0.649 °C

El factor de Van’t Hoff será:

2.769

m 0.126m

C1.86

C 0.649

i

La disociación porcentual se calcula considerando la ecuación:

1

i

los iones formados, como se observa en la ecuación de disociación es: = 4, reemplazando en

la anterior expresión:

0.589714

12.769α

La disociación porcentual es:

= 0.5897 100% = 58.97%

3.25 Al mezclar 17.6 kg del anticongelante comercial (etilenglicol, C2H6O2) con 6 galones de

agua, se abate su punto de congelación a –10.0 °F. Suponga que se desea lograr el mismo

efecto con sacarosa, C12H22O11 (que es una mala idea), en vez de etilenglicol. ¿Cuántos

kilogramos de sacarosa necesitaría disolver? 1 galón = 3.785 litros.

Solución: En principio calcularemos la molalidad del etilenglicol, puesto que hay que abatir el

punto de congelación a – 10.0 °F las molalidades deben ser idénticas, es decir:

a partir de la molalidad ha sido posible determinar la masa en kg de sacarosa para abatir la

temperatura de congelación, esto es se necesitan 97.09 kg de sacarosa.

112212

2

2

2

2

112212

112212

112212

112212

2

112212

OHC kg 97.096gal

OH gal 1

OH 3.785

OH 1

OH kg 1

OHC g 000 1

OHC kg 1

OHC mol 1

OHC g 342

OH kg 1

OHC moles12.5

molal 12.5OHC g 62

OHC mol 1

OHC kg 1

OHC g 1000

1

kg 1

gal 1

3.785OH gal 6

OHC kg 17.6m

262

262

262

262

2

262

Page 46: Soluciones.y.propiedades

SOLUCIONES Y PROPIEDADES COLIGATIVAS

COMPENDIOS DE QUÍMICA GENERAL PARA ESTUDIANTES DE CIENCIAS E INGENIERÍA

102

PROBLEMAS PROPUESTOS

(Soluciones)

3.1 a) Calcular la cantidad de sosa caústica y de agua que se necesita para

preparar 5 litros de una solución al 20%, cuya densidad es 1.219 g/ml, b) Cual

es la normalidad de está disolución?, c) Cuál es la molalidad?

Rpta.- 1219 g NaOH y 4876 g H2O; 6.095 N

3.2 Calcular el volumen de disolución al 12% en peso de CuSO4, que podrá prepararse con 1

kg de sulfato cúprico cristalizado, (CuSO45H2O). La densidad de la solución es 1.131 g/ml.

Rpta.- 4.71 litros

3.3 Determinar la masa de sulfato magnésico heptahidratado, (MgSO47H2O), que debe

añadirse a 1.5 de agua para obtener una disolución al 20% en masa de sulfato anhídro.

3.3 Se disuelven X g de MgCl2 en agua formando 2 litros de solución, observándose una

concentración de 24 mg de Mg+2

en cada cm3 de solución. Hallar el valor de X.

Rpta. 190

3.5 Se disuelven 130 g de una base de metal monovalente desconocida en agua formándose 1

litro de solución. Hallar su normalidad si se sabe que 0.78 g de la base produce 0.03 moles de

(OH)-1

.

Rpta. 5 normal

3.7 Calcular el volumen de una disolución de ácido sulfúrico de densidad 1.827 g/ml y

92.77% de H2SO4 que se necesita para preparar 10 litros de ácido sulfúrico 3 normal.

Rpta.- 868.2 ml

3.8 Determinar el volumen de una disolución 2 normal de sulfato cúprico que se necesita para

preparar 10 g de óxido cúprico previa precipitación del cobre como carbonato y calcinación

posterior de éste a óxido. Considerar las siguientes reacciones:

Na2CO3 + CuSO4 → CuCO3 + Na2SO4

CuCO3 + calor → CuO + CO2

3.9 ¿Qué volumen de ácido nítrico diluido, de densidad 1.11 g/ml y al 19% en masa de HNO3,

contiene 10 g HNO3 ?

3.10 Calcúlese la molalidad de una solución que contiene a) 0.65 moles de glucosa, en 250 g

de agua, b) 45 g de glucosa en 1 kg de agua, c) 18 g de glucosa en 1 libra de agua.

Rpta.- a) 2.6 m, b) 0.25 m, c) 0.30 m

3.11 Una solución acuosa etiquetada muestra una concentración del 35% en masa de HClO4,

una densidad de 1.251 g/cc. ¿Cuál es la concentración molar y la molalidad de la solución?

Page 47: Soluciones.y.propiedades

SOLUCIONES Y PROPIEDADES COLIGATIVAS

COMPENDIOS DE QUÍMICA GENERAL PARA ESTUDIANTES DE CIENCIAS E INGENIERÍA

103

Rpta.- 4.36 M; 5.36 m

3.12 El agua y el metanol son miscibles en todas las proporciones, si se mezclan 16 g de

metanol con 27 g de agua. ¿Cuál es la fracción molar del metanol en la solución?

3.13 Realice las siguientes conversiones siguientes:

a) Ca(OH)2 0.1 N en molaridad

b) 20 mg CuSO3/ml en molaridad y normalidad

c) 2.4 mg CaCl2/ml en molaridad y normalidad

d) 1.2 molal de nitrato de plata en tanto por ciento en masa

e) 2 N de H3PO4 en molaridad

3.14 Una reacción requiere 3.4 milimoles de Na3PO4. ¿Cuántos mililitros de una disolución

1.8 N se usarían?

3.15 Se mezclan 150 cc de K2SO4 3 M con 80 cc de NaNO3 2M. ¿Cuál es la concentración de

cada sal en la disolución al final?

3.16 ¿Cuántos mililitros de H2SO4 3.0 M se necesitan para neutralizar 200 ml de 0.34 N de

Ca(OH)2 ? ¿Cuántos miliequivalentes de CaSO4 se formaran?

3.17 El ácido clorhídrico concentrado tiene una concentración de 37% en masa de HCl y tiene

una densidad relativa de 1.184. Hallar la molaridad, normalidad y molalidad.

3.18 Se titularon exactamente 50 cc de una disolución de Na2CO3 con 65.8 cc de HCl 3.0 N

de acuerdo a:

Na2CO3 + 2HCl → NaCl + CO2 + H2O

Si la densidad de la solución de Na2CO3 es 1.25 g/cc. ¿Qué porcentaje de Na2CO3 en masa

contiene?

Rpta.- 16.7%

3.19 a) Que volumen de K2Cr2O7 0.40 N se necesita para liberar el cloro de 1.2 g de NaCl en

una solución acidifícada con H2SO4 ?

Cr2O7= + Cl

- + H

+→ Cr

+3 + Cl2 + H2O

b) ¿Cuántos gramos de cloro gaseoso se liberan?

Rpta.- 51 ml; 0.73 g

3.20 ¿Qué volumen de una disolución de perclorato férrico 0.5 N se necesitan para preparar

25 g de cloruro férrico?, en una solución básica de hidróxido plumboso, considerando la

siguiente reacción química.

Pb(OH)2 + Fe(ClO4)3 → FeCl3 + Pb3O4 + H2O

3.21 Si 600 cc de una solución 1.1 N de HCl son diluidos hasta formar exactamente la

solución 1 normal. ¿Qué cantidad de agua ha sido adicionada?

Page 48: Soluciones.y.propiedades

SOLUCIONES Y PROPIEDADES COLIGATIVAS

COMPENDIOS DE QUÍMICA GENERAL PARA ESTUDIANTES DE CIENCIAS E INGENIERÍA

104

3.22 Una solución de NaOH es 0.9 N. ¿Qué cantidad de la misma será necesaria para preparar

un litro de solución 0.1 N?

3.23 40 ml de solución 0.5 N de HCl se mezclan con 30 cc de solución normal de H2SO4.

¿Cuántos ml de solución 0.333 N de NaOH serán necesarios para neutralizar la mezcla de

ácidos?

3.24 Una muestra de un hidróxido alcalino puro (NaOH o KOH) es disuelta en agua y

requiere 50 cc de solución ácida 0.5 N para su neutralización. ¿De cuál de las dos bases

indicadas era la muestra?

3.25 Una muestra de 1.5 g de un metal puro ha necesitado 45.9 cc de solución normal de HCl

para ser disuelta. El metal es bivalente; calcular su peso atómico aproximado.

3.26 Cuantos gramos de hierro se disuelven en medio litro de solución 0.1 N de HCl?

3.27 10 g de una solución acuosa que contiene solamente Na2SO4 y H2SO4 se valora con

solución 0.5 normal de NaOH, de la cual se gastan 24.26 cc para la neutralización; la solución

resultante es tratada con BaCl2 en exceso y se obtiene 3.1526 g de BaSO4. Calcular el

porcentaje de a) H2SO4 y b) Na2SO4 en la solución original.

3.28 Si 51 cc de solución 0.1 N de HCl son tratados con 49 cc de solución 0.1 N de NaOH,

calcular: a) la normalidad en ácido de la solución resultante; b) el número de gramos de NaCl

que dejará la solución al ser evaporada a sequedad.

3.29 La solución de HCl de peso específico relativo 1.9545 contiene 10.94% de HCl en masa,

¿Cuántos ml son necesarios para preparar dos litros de solución 2 N?

3.30 Cuántos litros de gas NH3 seco medido a 27 ºC y 733 mmHg , serán necesarios para

preparar dos litros de solución exactamente ½ N de NH4OH.

3.31 25 cc de solución 0.5 N de H2SO4 son agregados a 40 cc de solución 0.25 N de NaOH.

¿Cuál está en exceso y en que cantidad?. Expresar la normalidad de la solución resultante en

función de a) el ácido o la base en exceso; b) el ión sulfato.

3.32 Calcular la pureza en tanto por ciento de una muestra de H2C2O42H2O partiendo del

dato de que 1.00 g de la misma es neutralizado por 31.4 cc de solución básica 0.5 N.

3.33 Si 1 g de H2C2O42H2O puro neutraliza 33.0 cc de una solución de KOH, calcular la

concentración de ésta en: a) gramos de KOH por litro; b) normalidad como bases; c)

molaridad.

3.34 La solución de NH4OH de peso específico relativo 0.954 contiene 11.64% de NH3.

Calcular su normalidad como base.

3.35 Masa iguales de NaOH y KOH son disueltos separadamente en la misma cantidad de

agua. ¿Cuál es la relación de sus normalidades?.

Page 49: Soluciones.y.propiedades

SOLUCIONES Y PROPIEDADES COLIGATIVAS

COMPENDIOS DE QUÍMICA GENERAL PARA ESTUDIANTES DE CIENCIAS E INGENIERÍA

105

3.36 Calcular la normalidad de una solución de K2Cr2O7 sabiendo que 48 cc de la misma

oxidan 1.375 g de FeSO47H2O puro.

3.37 Una solución de H2SO4 de peso específico relativo 1.160 contiene 22.25% en masa de

H2SO4. Calcular: a) su normalidad; b) su molaridad, c) su molalidad, d) su fracción molar.

3.38 Leemos en un libro de prácticas de química: Si se miden 40 ml de HCl concentrado

(densidad 1.195 g/ml, 38.72% HCl) y se diluye hasta formar un litro de solución, ésta resulta

aproximadamente 0.5 N. Háganse los cálculos para ver si esto es verdad.

Rpta. 0.507 N

3.39 Calcular la normalidad de una solución concentrada de H2SO4 según los datos siguientes,

densidad 1.8068 g/ml y 87.81% en masa

3.40 Se desea preparar exactamente 10 litros de solución 0.750 N de NaOH. ¿Cuántos

mililitros de solución de sosa cáustica de densidad 1.160 y 14.45% en masa de NaOH deben

emplearse?.

3.41 Para preparar 10 litros exactos de solución 0.111 N de KOH, disponemos de 40.22 g de

KOH puro. ¿Cuántos ml de solución de potasa cáustica, de peso específico 1.3010 del 31% en

masa de KOH son necesarios para completar la solución?

3.42 Se disuelve 350 g de cloruro de cinc anhidro (densidad relativa 2.91) en 650 g de agua,

se obtiene una disolución cuyo volumen total, a 20 ºC es de 740 ml. Calcular: a) la molaridad,

b) la normalidad, c) la molalidad, d) las fracciones molares, e) el tanto por ciento en masa

Rpta. 3.47 M, 6.94 N, 3.95 m, 0.0665, 35% ZnCl

(Propiedades Coligativas)

3.43 A la temperatura de 65 ºC hallar la presión de vapor sobre una disolución que contiene

13.68 g de sacarosa en 90 g de agua, si la presión de vapor saturado sobre el agua a la misma

temperatura es igual a 25.0 kPa (187.5 mm Hg).

3.44 ¿A que es igual la presión de vapor saturado sobre una disolución al 10% de carbamida

CO(NH2)2 a 100 ºC?

3.45 A la temperatura de 315 K, la presión de vapor saturado sobre el agua es igual a 8.2 kPa

(61.5 mmHg). ¿Cuanto disminuirá la presión de vapor a la temperatura señalada, si en 540 g

de agua se disuelven 36 g de glucosa?

3.46 A 293 K la presión de vapor saturado sobre el agua es igual a 2.34 kPa (17.53 mmHg).

¿Cuántos gramos de glicerina C3H5(OH)3 se deben disolver en 180 g de agua para disminuir

la presión de vapor en 133.3 Pa?

3.47 ¿Cuántos grados aumentará el punto de ebullición del agua si en 100 g de ésta se

disuelven 9 g de glucosa?

3.48 ¿A que temperatura, aproximadamente, hervirá la disolución al 50% en masa de

sacarosa?

Page 50: Soluciones.y.propiedades

SOLUCIONES Y PROPIEDADES COLIGATIVAS

COMPENDIOS DE QUÍMICA GENERAL PARA ESTUDIANTES DE CIENCIAS E INGENIERÍA

106

3.49 ¿A que temperatura, aproximadamente, cristalizará la disolución al 40% en masa de

alcohol etílico?.

3.50 ¿Cuántos gramos de sacarosa se deben disolver en 100 g de agua para: a) disminuir la

temperatura de cristalización en 1 grado; b) aumentar la temperatura de ebullición en 1 grado?

3.51. ¿En que relación deben encontrarse las masas de agua y de alcohol etílico para que al

mezclarlos, se obtenga una disolución que se cristalice a –20 ºC?.

3.52 En el radiador de automóvil vertieron 9 litros de agua y añadieron 2 litros de alcohol

metílico ( = 0,8 g/ml. Hecho esto, ¿a que temperatura mínima se puede dejar el coche al aire

libre sin temer que el agua en el radiador se congele?

3.53 Al disolver 5.0 g de sustancia en 200 g de agua se obtiene una disolución no conductora

de corriente, la cual se cristaliza a -1.45 ºC. Determine el peso molecular del soluto.

3.54 Al disolver 13 g de no electrolito en 400 g de éter dietílico (C2H5)2O, la temperatura de

ebullición se elevó en 0.453 K. Determinar el peso molecular del soluto.

3.55 En 60 g de benceno están disueltos 2.09 g de cierta sustancia cuya composición

elemental en masa es como sigue: 50.69% de C; 4.23% de H y 45.08% de O. La disolución se

cristaliza a 4.25 ºC. Establecer la fórmula molecular de la sustancia. El benceno puro se

cristaliza a 5.5 ºC

3.56 Una disolución acuoso-alcohólica que contiene 15% de alcohol desconocido ( = 0.97

g/ml) se cristaliza a -10.26 ºC. Hallar el peso molecular del alcohol desconocido y la presión

osmótica de la disolución a 293 K.

3.57 La temperatura de ebullición de la disolución acuosa de sacarosa es igual a 101.4 ºC.

Calcular la concentración molal y el tanto por ciento en masa de la sacarosa en la disolución.

¿A que temperatura se congela esta disolución?

3.58 ¿Cuántos gramos de glucosa C6H12O6 deben encontrarse en 0.5 litros de disolución para

que su presión osmótica (a igual temperatura) sea la misma que la de una disolución, 1 litro de

la cual contiene 9.2 g de glicerina C3H5(OH)3?

3.59 A 25 ºC la presión osmótica de la disolución cuyos 200 ml contiene 2.80 g de compuesto

macromolecular es igual a 0.70 kPa. Hallar el peso molecular del soluto.

3.60 A la temperatura de 20 ºC y 1 litro de disolución de un no electrolito cuya presión

osmótica es de 243.4 kPa se mezcla con 3 litros de disolución de no electrolito cuya presión

osmótica es igual a 486.8 kPa. Determinar la presión osmótica de la disolución mezclada

3.61 Una disolución en cuyos 100 ml se encuentran 2.3 g de cierta sustancia presenta, a 298

K, una presión osmótica igual a 618.5 kPa. Determinar el peso molecular de la sustancia

3.62 Un ml de disolución contienen 1018

moléculas de no electrolito disuelto. Calcular la

presión osmótica de la disolución a 298 K.

Page 51: Soluciones.y.propiedades

SOLUCIONES Y PROPIEDADES COLIGATIVAS

COMPENDIOS DE QUÍMICA GENERAL PARA ESTUDIANTES DE CIENCIAS E INGENIERÍA

107

3.63 La presión de vapor del agua pura a 25 ºC es de 23.76 mmHg, ¿Cuál es la presión de

vapor de una disolución que contiene 12 g de glucuosa, en 50 g de agua ?

3.64 Se disuelven 24.63 g de glucosa en 150 g de agua. A 23 ºC la presión de vapor de la

disolución es de 20.73 mmHg y la del agua pura es de 21.07 mmHg. ¿Qué peso molecular

tiene la glucosa?

3.65 A 30 ºC, la presión de vapor del eter dietílico es de 646 mmHg y la de la acetona pura de

283 mmHg.. Calcule la composición de una mezcla cuya presión sea 460 mmHg, suponiendo

la idealidad.

3.66 A 11 ºC las presiones de vapor del clorobenceno, C6H5Cl y del bromobenceno C6H5Br

son, respectivamente de 400 y 200 mmHg. Determinar la presión de vapor a esta temperatura

de una mezcla líquida supuesta ideal, formada por un 30% de C6H5Cl y 70% de C6H5Br, en

masa.

3.67 Suponga que se disuelven 5.0 gramos de una mezcla de naftaleno y antraceno, en 300

gramos de benceno. Se observa que la disolución se congela a 4.85 ºC. Encuentre la

composición porcentual (en masa) de la mezcla.

3.68 Al mezclar 17.6 kg del anticongelante comercial (etilenglicol, C2H6O2) con 6 galones de

agua, se abate su punto de congelación a –10 ºF. Suponga que se desea lograr el mismo efecto

en sacarosa, C12H22O11 ( que es una mala idea), en vez de etilenglicol. ¿Cuántos kg de

sacarosa necesitaría disolver? ( 1 galón = 3.785 )

3.69 El calor latente de fusión del cloruro mercúrico (HgCl2) es de 16.9 cal/g en su punto de

fusión de 265 °C. Al colocar en 50 g de aquél, que actúa como disolvente, 0.849 g de cloruro

mercuroso que actúa como soluto, el punto de fusión de la solución desciende en 1.24 °C.

Estimar el peso molecular del cloruro mercuroso y su fórmula molecular.

Rpta.- 236.1; Hg2Cl2

3.70 Una disolución de 0.52 g de cloruro potásico en 83.6 g de agua congela a - 0.291 °C.

Calcular el factor de van’t Hoff, el coeficiente osmótico y el grado de disociación aparente del

KCl. Kc(H2O) = 1.86 °C/mol.

Rpta. i = 1.876; g = 0.938; = 0.876

3.71 El punto de ebullición de una disolución de 3.41 g de cloruro cálcico en 100 g de agua es

100.21 °C. Calcular el factor de van’t Hoff, el coeficiente osmótico y el grado de disociación

aparente del cloruro de bario. Ke(H2O) = 0.52 °C/mol

Rpta. i = 2.466; g = 0.822; = 0.733

3.72 Una disolución de cloruro de cinc al 1%, y de densidad prácticamente la unidad, congela

a – 0.28 °C. Calcular el grado de disociación aparente del cloruro de cinc, y a partir de él, la

concentración de los iones en la disolución.

Rpta. = 0.516; [Zn+2

] = 0.0375 M

Page 52: Soluciones.y.propiedades

SOLUCIONES Y PROPIEDADES COLIGATIVAS

COMPENDIOS DE QUÍMICA GENERAL PARA ESTUDIANTES DE CIENCIAS E INGENIERÍA

108

3.73 Calcular la presión de vapor a 100 °C de una disolución de cloruro de sodio al 10%,

suponiendo que el grado de disociación aparente de la sal sea del 90%.

Rpta. 713.6 mmHg

3.74 Una disolución de cloruro potásico que contiene 1 g de sal por litro ejerce , a 14 °C, una

presión osmótica de 456 mmHg. Calcular el valor del coeficiente osmótico y el grado de

disociación aparente del cloruro potásico.

Rpta. g = 0.951; = 90.1%

3.75 Calcular la presión osmótica a 18 °C de una solución de cloruro sódico que contiene 2.5

g de sal en 100 cm3, si el factor de van’ Hoff es igual a 1.83.

Rpta. 18.68 atm

3.76 Un cloruro hidratado de calcio contiene 27.21% de Ca y 28.30% de Cl. La disolución

formada al disolver 0.364 g del compuesto en 50 g de agua empieza a congelar a – 0.29 °C. a)

Escribir la fórmula racional del compuesto, b) determine el factor de van’t Hoff. Kc(H2O) =

1.86 °C/mol

Rpta. CaCl2H2O; i = 3.12

3.77 Una solución acuosa con 5.00% en peso de ácido sulfúrico, tiene un punto de

congelación de – 2.17 °C. Calcular:

a) El factor de Van’t Hoff.

b) El coeficiente osmótico de la solución

Rpta.- 2.17; 0.723

3.78 Una solución acuosa 0.100 molal de ácido acético, presenta disociado el CH3COOH en

1.35%. Calcular:

a) El punto de congelación de la solución

b) La presión osmótica de la solución

Rpta.- -0.188 °C; 2.48 atm

3.79 Se ha disuelto 160 g de ácido oxálico en 1000 g de agua, resultando que la solución

hierve a 102.41 °C, a la presión de 1 atm. Calcular el grado de disociación del COOH –

COOH.

Rpta.- 0.82

3.80 El descenso de la presión de vapor producido por un mol de NaCl en 1000 g de agua a 18

°C, es de 0.475 mmHg. La tensión de vapor de agua a 18 °C es de 15.477 mmHg. Calcular:

a) El factor de Van’t Hoff.

b) La presión osmótica

Rpta.- 1.734; 41.38 atm